Как найти показатель матрицы

Определитель матрицы: алгоритм и примеры вычисления определителя матрицы

Перед тем как находить и считать определитель, дадим определение определителю матрицы. 

Определение 1

Что такое определитель матрицы или детерминант матрицы? Определитель матрицы — это некоторое число, с которым можно сопоставить любую квадратную матрицу А=(aij)n×n. 

|А|, ∆, det A – символы, которыми обозначают определитель матрицы.

Как найти определитель матрицы? Вычислить определитель или найти определитель можно с помощью разных способов (в том числе онлайн и при помощи калькулятора). Конкретный способ поиска и того, как решать, выбирают в зависимости от порядка матрицы.

Пример 1​​​​​

Определитель матрицы второго порядка можно вычислять по формуле:

А=1-231.

Решение матрицы:

det A=1-231=1×1-3×(-2)=1+6=7

Определитель матрицы 3-го порядка: правило треугольника 

Нахождение определителя матрицы 3-го порядка осуществляется по одному из правил:

  • он может считаться по правилу треугольника;
  • расчет также проводится по правилу Саррюса.

Как найти определитель матрицы третьего порядка по методу треугольника (определитель матрицы 3×3)?

а11а12а13а21а22а23а31а32а33=a11×a22×a33+a31×a12×a23+a21×a32×a13-a31×a22×a13-a21×a12×a33-a11×a23×a32

Пример 2

А=13402115-1

Решение:

det A=13402115-1=1×2×(-2)+1×3×1+4×0×5-1×2×4-0×3×(-1)-5×1×1=(-2)+3+0-8-0-5=-12

Правило Саррюса

Чтобы вычислить определитель по методу Саррюса, необходимо учесть некоторые условия и выполнить следующие действия:

  • дописать слева от определителя два первых столбца;
  • перемножить элементы, которые расположены на главной диагонали и параллельных ей диагоналях, взяв произведения со знаком «+»;
  • перемножить элементы, которые расположены на побочных диагоналях и параллельных им, взяв произведения со знаком «—».

а11а12а13а21а22а23а31а32а33=a11×a22×a33+a31×a12×a23+a21×a32×a13-a31×a22×a13-a21×a12×a33-a11×a23×a32

Пример 3

А=134021-25-11302-25=1×2×(-1)+3×1×(-2)+4×0×5-4×2×(-2)-1×1×5-3×0×(-1)=-2-6+0+16-5-0=3

Методы разложения по элементам строки и столбца

Чтобы вычислить определитель матрицы четвертого порядка, можно воспользоваться одним из 2-х способов:

  • разложением по элементам строки;
  • разложением по элементам столбца.

Представленные способы определяют вычисление определителя n как вычисление определителя порядка n-1 за счет представления определителя суммой произведений элементов строки (столбца) на их алгебраические дополнения.

Пример 4

Разложение матрицы по элементам строки:

det A=ai1×Ai1+ai2×Ai2+…+аin×Аin

Разложение матрицы по элементам столбца:

det A=а1i×А1i+а2i×А2i+…+аni×Аni

Замечание

Если раскладывать матрицу по элементам строки (столбца), необходимо выбирать строку (столбец), в которой(-ом) есть нули.

Пример 5

А=01-132100-24513210

Решение:

  • раскладываем по 2-ой строке:

А=01-132100-24513210=2×(-1)3×1-13-251310=-2×1-13451210+1×0-13-251310

  • раскладываем по 4-му столбцу:

А=01-132100-24513210=3×(-1)5×210-245321+1×(-1)7×01-1210321=-3×210-245321-1×01-1210321

Свойства определителя

Свойства определителя:

  • если преобразовывать столбцы или строки незначительными действиями, то это не влияет на значение определителя;
  • если поменять местами строки и столбцы, то знак поменяется на противоположный;
  • определитель треугольной матрицы представляет собой произведение элементов, которые расположены на главной диагонали.
Замечание 

В рамках темы советуем обратиться к модулю определителя.

Пример 6

А=134021005

Решение:

det А=134021005=1×5×2=10

Замечание

Матричныый определитель, который содержит нулевой столбец, равный нулю (представляет собой минор).

Ирина Мальцевская

Преподаватель математики и информатики. Кафедра бизнес-информатики Российского университета транспорта

Определи́тель (детермина́нт) в линейной алгебре — скалярная величина, которая характеризует ориентированное «растяжение» или «сжатие» многомерного евклидова пространства после преобразования матрицей; имеет смысл только для квадратных матриц. Стандартные обозначения определителя матрицы A{displaystyle det(A)}, |A|, {displaystyle Delta (A)}[1].

Определитель квадратной матрицы A размеров ntimes n, заданной над коммутативным кольцом R, является элементом кольца R. Эта величина определяет многие свойства матрицы A, в частности, матрица обратима тогда и только тогда, когда её определитель является обратимым элементом кольца R. В случае, когда R — поле, определитель матрицы A равен нулю тогда и только тогда, когда ранг матрицы A меньше n, то есть когда системы строк и столбцов матрицы A являются линейно зависимыми.

История[править | править код]

Теория определителей возникла в связи с задачей решения систем линейных уравнений.

К понятию определителя близко подошли авторы древнекитайского учебника «Математика в девяти книгах»[2].

В Европе определители матриц 2 × 2 встречаются у Кардано в XVI веке.
Для старших размерностей определение детерминанта дано Лейбницем в 1693 году.
Первая публикация принадлежит Крамеру. Теория определителей создана Вандермондом, Лапласом, Коши и Якоби. Термин «определитель» в современном его значении ввёл О. Коши (1815), хотя ранее (1801) «детерминантом» К. Гаусс назвал дискриминант квадратичной формы.

Японский математик Сэки Такакадзу ввёл определители независимо в 1683 году[3].

Определения[править | править код]

Через перестановки[править | править код]

Для квадратной матрицы A=(a_{ij}) размера ntimes n её определитель {displaystyle det A} вычисляется по формуле:

{displaystyle det A=sum _{alpha _{1},alpha _{2},ldots ,alpha _{n}}(-1)^{N(alpha _{1},alpha _{2},ldots ,alpha _{n})}cdot a_{1alpha _{1}}a_{2alpha _{2}}dots a_{nalpha _{n}}},

где суммирование проводится по всем перестановкам {displaystyle alpha _{1},alpha _{2},ldots ,alpha _{n}} чисел 1,2,dots ,n, а {displaystyle N(alpha _{1},alpha _{2},ldots ,alpha _{n})} обозначает число инверсий в перестановке {displaystyle alpha _{1},alpha _{2},ldots ,alpha _{n}}.

Таким образом, в определитель входит n! слагаемых, которые также называют «членами определителя».

Эквивалентная формула:

{displaystyle det A=sum _{i_{1},i_{2},ldots ,i_{n}=1}^{n}varepsilon _{i_{1}i_{2}dots i_{n}}cdot a_{1i_{1}}a_{2i_{2}}dots a_{ni_{n}}},

где коэффициент {displaystyle varepsilon _{i_{1}i_{2}dots i_{n}}} — символ Леви-Чивиты — равен:

0, если не все индексы {displaystyle i_{1},i_{2},ldots ,i_{n}} различны,
1, если все индексы {displaystyle i_{1},i_{2},ldots ,i_{n}} различны и подстановка {displaystyle {begin{pmatrix}1&2&dots &n\i_{1}&i_{2}&dots &i_{n}end{pmatrix}}in S_{n}} чётна,
−1, если все индексы {displaystyle i_{1},i_{2},ldots ,i_{n}} различны и подстановка {displaystyle {begin{pmatrix}1&2&dots &n\i_{1}&i_{2}&dots &i_{n}end{pmatrix}}in S_{n}} нечётна.

Аксиоматическое построение (определение на основе свойств)[править | править код]

Понятие определителя может быть введено на основе его свойств. А именно, определителем вещественной матрицы называется функция det :mathbb {R} ^{ntimes n}rightarrow mathbb {R} , обладающая следующими тремя свойствами[4]:

  1. det(A) — кососимметрическая функция строк (столбцов) матрицы A.
  2. det(A) — полилинейная функция строк (столбцов) матрицы A.
  3. det(E)=1, где E — единичная ntimes n-матрица.

Значение определителя матрицы[править | править код]

Для матрицы первого порядка значение детерминанта равно единственному элементу этой матрицы:

Delta ={begin{vmatrix}a_{11}end{vmatrix}}=a_{11}

Матрицы 2 x 2[править | править код]

Схема расчета определителя матрицы 2×2.

Площадь параллелограмма равна модулю определителя матрицы, образованной векторами — сторонами параллелограмма.

Для матрицы 2times 2 определитель вычисляется как:

Delta ={begin{vmatrix}a&c\b&dend{vmatrix}}=ad-bc

Эта матрица A может быть рассмотрена как матрица линейного отображения, преобразующего единичный квадрат в параллелограмм с вершинами (0, 0), (a, b), (a + c, b + d), и (c, d).

Абсолютное значение определителя |ad-bc| равно площади этого параллелограмма, и, таким образом, отражает коэффициент, на который масштабируются площади при преобразовании A.

Значение определителя со знаком (ориентированная площадь параллелограмма) помимо коэффициента масштабирования также показывает, выполняет ли преобразование A отражение.

Матрицы 3 x 3[править | править код]

Определитель матрицы 3times 3 можно вычислить по формуле:

Delta ={begin{vmatrix}a_{11}&a_{12}&a_{13}\a_{21}&a_{22}&a_{23}\a_{31}&a_{32}&a_{33}end{vmatrix}}=a_{11}{begin{vmatrix}a_{22}&a_{23}\a_{32}&a_{33}end{vmatrix}}-a_{12}{begin{vmatrix}a_{21}&a_{23}\a_{31}&a_{33}end{vmatrix}}+a_{13}{begin{vmatrix}a_{21}&a_{22}\a_{31}&a_{32}end{vmatrix}}=
=a_{11}a_{22}a_{33}-a_{11}a_{23}a_{32}-a_{12}a_{21}a_{33}+a_{12}a_{23}a_{31}+a_{13}a_{21}a_{32}-a_{13}a_{22}a_{31}

Для более удобного вычисления определителя третьего порядка можно воспользоваться правилом Саррюса или правилом треугольника.

Определитель матрицы, составленной из векторов mathbf {a} ,mathbf {b} ,mathbf {c} равен их смешанному произведению в правой декартовой системе координат и, аналогично двумерному случаю, представляет собой ориентированный объём параллелепипеда, натянутого на mathbf {a} ,mathbf {b} ,mathbf {c} . Геометрически определитель матрицы 3х3 означает объем параллелепипеда, построенного на векторах матрицы. [5]

Матрицы N × N[править | править код]

В общем случае, для матриц более высоких порядков (выше 2-го порядка) ntimes n определитель можно вычислить, применив следующую рекурсивную формулу:

Delta =sum _{j=1}^{n}(-1)^{1+j}a_{1j}{bar {M}}_{j}^{1}, где {bar {M}}_{j}^{1} — дополнительный минор к элементу a_{1j}. Эта формула называется разложением по строке.

Легко доказать, что при транспонировании определитель матрицы не изменяется (иными словами, аналогичное разложение по первому столбцу также справедливо, то есть даёт такой же результат, как и разложение по первой строке):

Delta =sum _{i=1}^{n}(-1)^{i+1}a_{i1}{bar {M}}_{1}^{i}

Также справедливо и аналогичное разложение по любой строке (столбцу):

Delta =sum _{j=1}^{n}(-1)^{i+j}a_{ij}{bar {M}}_{j}^{i}

Обобщением вышеуказанных формул является разложение детерминанта по Лапласу (Теорема Лапласа), дающее возможность вычислять определитель по любым k строкам (столбцам):

{displaystyle Delta =sum _{1leqslant j_{1}<ldots <j_{k}leqslant n}(-1)^{i_{1}+ldots +i_{k}+j_{1}+ldots +j_{k}}M_{j_{1}ldots j_{k}}^{i_{1}ldots i_{k}}{bar {M}}_{j_{1}ldots j_{k}}^{i_{1}ldots i_{k}}}

Альтернативные методы вычисления[править | править код]

  • Метод конденсации Доджсона, основанный на рекурсивной формуле:
{displaystyle Delta ={frac {{bar {M}}_{1}^{1}{bar {M}}_{k}^{k}-{bar {M}}_{1}^{k}{bar {M}}_{k}^{1}}{{bar {M}}_{1,k}^{1,k}}}}.

Основные свойства определителей[править | править код]

Следующие свойства отражают основные результаты теории определителей, применение которых выходит далеко за пределы этой теории:

  1. {displaystyle det E=1} (Определитель единичной матрицы равен 1);
  2. {displaystyle det cA=c^{n}det A} (Определитель является однородной функцией степени n на пространстве матриц размера ntimes n);
  3. {displaystyle det A^{T}=det A} (Определитель матрицы не меняется при её транспонировании);
  4. {displaystyle det(AB)=det Acdot det B} (Определитель произведения матриц равен произведению их определителей, A и B — квадратные матрицы одного и того же порядка);
  5. {displaystyle det A^{-1}=(det A)^{-1}}, причём матрица A обратима тогда и только тогда, когда обратим её определитель {displaystyle det A};
  6. Существует ненулевое решение уравнения AX=0 тогда и только тогда, когда det A=0 (или же {displaystyle det A} должен быть нетривиальным делителем нуля в случае, если R — не целостное кольцо).

Определитель как функция строк (столбцов) матрицы[править | править код]

При изучении теории определителей полезно иметь в виду, что в основе этой теории лежит техника манипулирования со строками и столбцами матриц, разработанная К.Ф. Гауссом (преобразования Гаусса). Суть этих преобразований сводится к линейным операциям над строками (столбцами) и их перестановке. Эти преобразования достаточно простым образом отражаются на определителе, и при их изучении удобно “расчленить” исходную матрицу на строки (или столбцы) и считать определитель функцией, определённой над наборами строк (столбцов). Далее буквами {displaystyle {hat {A}}_{i}} обозначаются строки (столбцы) матрицы A.

1. Определитель — полилинейная функция строк (столбцов) матрицы. Полилинейность означает линейность функции по каждому аргументу при фиксированных значениях остальных аргументов:

{displaystyle det({hat {A}}_{1},ldots ,alpha {hat {A}}_{i}+beta {hat {A'}}_{i},ldots ,{hat {A}}_{n})=alpha det({hat {A}}_{1},ldots ,{hat {A}}_{i},ldots ,{hat {A}}_{n})+beta det({hat {A}}_{1},ldots ,{hat {A'}}_{i},ldots ,{hat {A}}_{n})}
2. Определитель — кососимметрическая функция строк (столбцов) матрицы, то есть при перестановке двух строк (столбцов) матрицы её определитель умножается на −1:

{displaystyle det(ldots ,{hat {A}}_{i},ldots ,{hat {A}}_{j},ldots )=-det(ldots ,{hat {A}}_{j},ldots ,{hat {A}}_{i},ldots )}
3. Если две строки (столбца) матрицы совпадают, то её определитель равен нулю:

{displaystyle {hat {A}}_{i}={hat {A}}_{j}Longrightarrow det(ldots ,{hat {A}}_{i},ldots ,{hat {A}}_{j},ldots )=0}

Замечание. Свойства 1-3 являются основными свойствами определителя как функции строк (столбцов), они легко доказываются непосредственно из определения. Свойство 2 (кососимметричность) является логическим следствием свойств 1 и 3. Свойство 3 является логическим следствием свойства 2, если в кольце R элемент 2 (т.е. 1 + 1) не совпадает с нулём и не является делителем нуля. Из свойств 1 и 3 вытекают также следующие свойства:

4. Общий множитель элементов какой-либо строки (столбца) определителя можно вынести за знак определителя (следствие свойства 1).
5. Если хотя бы одна строка (столбец) матрицы нулевая, то определитель равен нулю (следствие свойства 4).
6. Если две (или несколько) строки (столбца) матрицы линейно зависимы, то её определитель равен нулю (следствие свойств 1 и 3).
7. При добавлении к любой строке (столбцу) линейной комбинации других строк (столбцов) определитель не изменится (следствие свойств 1 и 6).

Фактом, имеющим фундаментальное значение, является универсальность определителя как полилинейной кососимметрической функции полного ранга, аргументами которой являются элементы конечномерного векторного пространства V (или R-модуля V с конечным базисом). Справедлива следующая

Теорема. Пусть V — свободный R-модуль ранга n (n-мерное векторное пространство над R, если R — поле). Пусть {displaystyle varphi (X_{1},X_{2},dots ,X_{n})}R-значная функция на {displaystyle V^{n}}, обладающая свойствами 1-3. Тогда при выборе базиса {displaystyle E_{1},E_{2},dots ,E_{n}} пространства V существует константа {displaystyle c_{0}=varphi (E_{1},E_{2},dots ,E_{n})} такая, что при всех значениях {displaystyle X_{1},X_{2},dots ,X_{n}} справедливо равенство:

{displaystyle varphi (X_{1},X_{2},dots ,X_{n})=c_{0}det({hat {X}}_{1},{hat {X}}_{2},dots ,{hat {X}}_{n})},

где {displaystyle {hat {X}}_{i}} — столбец координат вектора X_{i} относительно базиса {displaystyle E_{1},E_{2},dots ,E_{n}}.

Одним из важнейших следствий универсальности определителя является следующая теорема о мультипликативности определителя.

Теорема. Пусть A — матрица размера ntimes n. Тогда {displaystyle det(AX)=det Acdot det X} для любой матрицы X размера ntimes n.

Доказательство

Рассмотрим на пространстве столбцов R^{n} кососимметрическую полилинейную форму {displaystyle varphi ({hat {X}}_{1},{hat {X}}_{2},dots ,{hat {X}}_{n})=det(A{hat {X}}_{1},A{hat {X}}_{2},dots ,A{hat {X}}_{n})=det(AX)}. Согласно доказанной теореме, эта форма равна {displaystyle c_{0}det({hat {X}}_{1},{hat {X}}_{2},dots ,{hat {X}}_{n})=c_{0}det X}, где {displaystyle c_{0}=varphi (E_{1},E_{2},dots ,E_{n})=det({hat {A}}_{1},{hat {A}}_{2},dots ,{hat {A}}_{n})=det A}.

Определитель и ориентированный объём[править | править код]

Пусть {vec {a}},{vec {b}},{vec {c}} — три вектора в пространстве mathbb {R} ^{3}. Они порождают параллелепипед, вершины которого лежат в точках с радиус-векторами {displaystyle {vec {0}},{vec {a}},{vec {b}},{vec {c}},{vec {a}}+{vec {b}},{vec {c}}+{vec {a}},{vec {b}}+{vec {c}},{vec {a}}+{vec {b}}+{vec {c}}}. Этот параллелепипед может быть вырожден, если вектора {vec {a}},{vec {b}},{vec {c}} компланарны (лежат в одной плоскости, линейно зависимы).

Функция ориентированного объёма {displaystyle {rm {Vol}}({vec {a}},{vec {b}},{vec {c}})} определяется как объём параллелепипеда, порождённого этими векторами, и взятый со знаком “+”, если тройка векторов {displaystyle ({vec {a}},{vec {b}},{vec {c}})} положительно ориентирована, и со знаком “-“, если она ориентирована отрицательно.
Функция {displaystyle {rm {Vol}}({vec {a}},{vec {b}},{vec {c}})} полилинейна и кососимметрична. Свойство 3, очевидно, выполнено. Для доказательства полилинейности этой функции, достаточно доказать её линейность по вектору vec a. Если вектора {displaystyle {vec {b}},{vec {c}}} линейно зависимы, значение {displaystyle {rm {Vol}}({vec {a}},{vec {b}},{vec {c}})} будет нулевым независимо от вектора vec a, и значит, линейно зависящим от него. Если вектора {displaystyle {vec {b}},{vec {c}}} линейно независимы, обозначим через {vec {n}} вектор единичной нормали к плоскости векторов {displaystyle {vec {b}},{vec {c}}}, такой, что {displaystyle {rm {Vol}}({vec {n}},{vec {b}},{vec {c}})>0}. Тогда ориентированный объём параллелепипеда равен произведению площади основания, построенного на векторах {displaystyle {vec {b}},{vec {c}}} и независящего от вектора vec a, и алгебраической величины проекции вектора vec a на нормаль к основанию, которая равна скалярному произведению {displaystyle langle {vec {a}},{vec {n}}rangle } и является величиной, линейно зависящей от вектора vec a. Линейность по vec a доказана, и аналогично доказывается линейность по остальным аргументам.

Применяя теорему об универсальности определителя как кососимметрической полилинейной функции, получаем, что при выборе ортонормированного базиса {displaystyle ({vec {e}}_{1},{vec {e}}_{2},{vec {e}}_{3})} пространства mathbb {R} ^{3}

{displaystyle {rm {Vol}}({vec {a}},{vec {b}},{vec {c}})={rm {Vol}}({vec {e}}_{1},{vec {e}}_{2},{vec {e}}_{3})det {begin{pmatrix}a_{1}&b_{1}&c_{1}\a_{2}&b_{2}&c_{2}\a_{3}&b_{3}&c_{3}end{pmatrix}}=det {begin{pmatrix}a_{1}&b_{1}&c_{1}\a_{2}&b_{2}&c_{2}\a_{3}&b_{3}&c_{3}end{pmatrix}}},

где a_{i},b_{i},c_{i} — координаты векторов {displaystyle ({vec {a}},{vec {b}},{vec {c}})} в выбранном базисе.

Таким образом, определитель матрицы коэффициентов векторов относительно ортонормированного базиса имеет смысл ориентированного объёма параллелепипеда, построенного на этих векторах.

Всё вышесказанное без существенных изменений переносится на пространство mathbb {R} ^{n} произвольной размерности.

Разложение определителя по строке/столбцу и обращение матриц[править | править код]

Формулы разложения определителя по строке/столбцу позволяют сводить вычисление определителей к рекурсивной процедуре, использующей вычисление определителей меньших порядков. Для вывода этих формул сгруппируем и просуммируем в формуле для определителя матрицы {displaystyle C=(c_{ij})}, с учётом равенства {displaystyle varepsilon _{i_{1}dots i_{n-1}n}=varepsilon _{i_{1}dots i_{n-1}}}, все ненулевые члены, содержащие элемент {displaystyle c_{nn}}. Эта сумма равна:

{displaystyle c_{nn}sum _{i_{1},dots i_{n-1}=1}^{n-1}varepsilon _{i_{1}dots i_{n-1}n}c_{1i_{1}}dots c_{n-1,i_{n-1}}=c_{nn}sum _{i_{1},dots i_{n-1}=1}^{n-1}varepsilon _{i_{1}dots i_{n-1}}c_{1i_{1}}dots c_{n-1,i_{n-1}}=c_{nn}det C_{n}^{n}},

где {displaystyle C_{i}^{j}} — матрица, получаемая из C удалением строки с номером i и столбца с номером j.

Так как произвольный элемент c_{ij} можно {displaystyle (n-j)} перестановками соответствующего столбца вправо и {displaystyle (n-i)} перестановками соответствующей строки вниз переместить в правый нижний угол матрицы, причём дополнительная к нему матрица сохранит свой вид, то сумма всех членов в разложении определителя, содержащих c_{ij}, будет равна

{displaystyle (-1)^{(n-i)}(-1)^{(n-j)}c_{ij}det C_{i}^{j}=(-1)^{i+j}c_{ij}det C_{i}^{j}=c_{ij}A_{i}^{j}}.

Величина {displaystyle A_{i}^{j}=(-1)^{i+j}det C_{i}^{j}} называется алгебраическим дополнением элемента матрицы c_{ij}.

Учитывая, что каждый член разложения определителя с ненулевым коэффициентом содержит ровно один элемент из i-ой строки, можно разложить определитель по членам этой строки:

{displaystyle det C=sum _{j=1}^{n}c_{ij}A_{i}^{j}} — Формула разложения определителя по i-ой строке

Аналогично, учитывая, что каждый член разложения определителя с ненулевым коэффициентом содержит ровно один элемент из j-го столбца, можно разложить определитель по членам этого столбца:

{displaystyle det C=sum _{i=1}^{n}c_{ij}A_{i}^{j}} — Формула разложения определителя по j-ому столбцу

Если элементы k-й строки матрицы C скопировать в i-ую строку, её определитель станет равен нулю, а по формуле разложения определителя по i-ой строке получится:

{displaystyle 0=sum _{j=1}^{n}c_{kj}A_{i}^{j}} — Формула “фальшивого” разложения определителя по i-ой строке ({displaystyle ineq k}).

Аналогично для столбцов:

{displaystyle 0=sum _{j=1}^{n}c_{ik}A_{i}^{j}} — Формула “фальшивого” разложения определителя по j-ому столбцу (jne k)

Полученные формулы полезно записать в матричном виде. Введём матрицу алгебраических дополнений к элементам матрицы C: {displaystyle A=(A_{i}^{j})}.
Тогда, согласно с полученными формулами,

{displaystyle CA^{T}=A^{T}C=(det C)cdot E}.

Следствие 1 (Критерий обратимости матриц). Квадратная матрица C является обратимой тогда и только тогда, когда {displaystyle det C} — обратимый элемент кольца R, при этом {displaystyle C^{-1}=(det C)^{-1}cdot A^{T}}.

Следствие 2. Если произведение матриц равно нулю {displaystyle CX=0}, и матрица C — квадратная, тогда {displaystyle (det C)cdot X=0}.

Решение систем линейных алгебраических уравнений с помощью определителей[править | править код]

Формула Крамера позволяет выразить решение системы линейных алгебраических уравнений в виде отношения определителей, в знаменателе которого стоит определитель системы, а в числителе — определитель матрицы системы, в которой столбец коэффициентов при соответствующей переменной заменён на столбец из правых частей уравнений.

Формула Крамера. Пусть задана система линейных алгебраических уравнений в матричном виде: {displaystyle CX=B}, где {displaystyle C=(c_{ij})} — матрица коэффициентов системы размера ntimes n, {displaystyle B=(b_{1},b_{2},dots ,b_{n})^{T}} — столбец из правых частей уравнений системы, и вектор {displaystyle X=(x_{1},x_{2},dots ,x_{n})^{T}} — решение этой системы. Тогда при любых {displaystyle lambda _{1},lambda _{2},dots ,lambda _{n}in R} справедливо равенство:

{displaystyle (lambda _{1}x_{1}+lambda _{2}x_{2}+dots +lambda _{n}x_{n})cdot (det C)=-det {begin{pmatrix}c_{11}&c_{12}&dots &c_{1n}&b_{1}\c_{21}&c_{22}&dots &c_{2n}&b_{2}\dots &dots &dots &dots &dots \c_{n1}&c_{n2}&dots &c_{nn}&b_{n}\lambda _{1}&lambda _{2}&dots &lambda _{n}&0end{pmatrix}}}

Из этой формулы следует, в частности, что если {displaystyle det C} — не вырожден (не является нулём или делителем нуля), система может обладать не более чем одним решением, а если определитель ещё и обратим, то система обладает единственным решением.

Одной из важнейших теорем в теории определителей является следующая теорема о решениях однородной системы линейных уравнений.

Теорема. Пусть R — поле. Однородная система линейных уравнений {displaystyle CX=0} имеет нетривиальное (ненулевое) решение тогда и только тогда, когда определитель матрицы коэффициентов равен нулю: {displaystyle det C=0}.

Доказательство

Необходимость условия содержится в следствии 2 предыдущего раздела. Докажем необходимость.

Если матрица C — нулевая, любой вектор X является решением.
Пусть C_{0} — максимальный невырожденный минор в матрице C имеет размеры rtimes r. Не ограничивая общности, считаем, что этот минор образован первыми r строками и столбцами (иначе перенумеруем переменные и переставим уравнения в другом порядке.)
Введём вектора {displaystyle X_{0}=(x_{1},x_{2},dots ,x_{r})^{T}} и {displaystyle X_{1}=(x_{r+1},dots ,x_{n})^{T}}. Тогда первые r уравнений системы в матричном виде записываются так:

{displaystyle C_{0}X_{0}+C_{1}X_{1}=0}

Поскольку матрица C_{0} обратима, любому значению X_{1} соответствует единственный вектор {displaystyle X_{0}=-C_{0}^{-1}C_{1}X_{1}}, удовлетворяющий этим уравнениям. Покажем, что при этом остальные уравнения будут выполнены автоматически. Пусть {displaystyle j>r}.

Введём две матрицы:

{displaystyle D_{1}={begin{pmatrix}c_{11}&dots &c_{1r}&c_{1r+1}x_{r+1}+dots +c_{1n}x_{n}\c_{21}&dots &c_{2r}&c_{2r+1}x_{r+1}+dots +c_{2n}x_{n}\dots &dots &dots &dots dots dots dots \c_{r1}&dots &c_{rr}&c_{rr+1}x_{r+1}+dots +c_{rn}x_{n}\c_{j1}&dots &c_{jr}&c_{jr+1}x_{r+1}+dots +c_{jn}x_{n}\end{pmatrix}}} и {displaystyle D_{2}={begin{pmatrix}c_{11}&dots &c_{1r}&c_{1r+1}x_{r+1}+dots +c_{1n}x_{n}\c_{21}&dots &c_{2r}&c_{2r+1}x_{r+1}+dots +c_{2n}x_{n}\dots &dots &dots &dots dots dots dots \c_{r1}&dots &c_{rr}&c_{rr+1}x_{r+1}+dots +c_{rn}x_{n}\c_{j1}&dots &c_{jr}&-(c_{j1}x_{1}+dots +c_{jr}x_{r})\end{pmatrix}}}.

У матрицы D_{1} все столбцы являются частями столбцов из матрицы C, а последний столбец есть линейная комбинация столбцов матрицы C с коэффициентами {displaystyle x_{r+1},dots ,x_{n}}, поэтому в силу линейности определителя по столбцам {displaystyle det D_{1}} есть линейная комбинация определителей миноров матрицы C размера {displaystyle (r+1)times (r+1)}. Так как C_{0} — максимальный по размеру невырожденный минор, все миноры большего размера имеют нулевой определитель, поэтому {displaystyle det D_{1}=0}.

Из соотношения {displaystyle C_{0}X_{0}+C_{1}X_{1}=0} следует, что {displaystyle D_{2}Y=0}, где столбец {displaystyle Y=(x_{1},dots ,x_{r},1)^{T}neq 0}. Поэтому {displaystyle det D_{2}=0}.

Тогда {displaystyle 0=det D_{1}-det D_{2}=(det C_{0})cdot (c_{j1}x_{1}+c_{j2}x_{2}+dots +c_{jn}x_{n})}. И так как {displaystyle det C_{0}neq 0}, то j-ое уравнение системы тоже выполнено.

Данная теорема используется, в частности, для нахождения собственных значений и собственных векторов матриц.

Критерий полноты и линейной независимости системы векторов[править | править код]

Тесно связанными с понятием определителя является понятие линейной зависимости и полноты систем векторов в векторном пространстве.

Пусть R — поле, V — векторное пространство над R с конечным базисом {displaystyle {vec {e_{1}}},{vec {e_{2}}},dots ,{vec {e_{n}}}}. Пусть задан ещё набор из n векторов {displaystyle {vec {v_{1}}},{vec {v_{2}}},dots ,{vec {v_{n}}}}. Их координатами {displaystyle v_{ij}} относительно заданного базиса являются коэффициенты разложения {displaystyle {vec {v_{i}}}=sum v_{ij}{vec {e_{j}}}}. Составим (квадратную) матрицу {displaystyle A=(v_{ij})}. Справедлива теорема:

Теорема (Критерий полноты и линейной независимости системы векторов).

(1) Система векторов {displaystyle {vec {v_{1}}},{vec {v_{2}}},dots ,{vec {v_{n}}}} линейно зависима тогда и только тогда, когда det A=0.
(2) Система векторов {displaystyle {vec {v_{1}}},{vec {v_{2}}},dots ,{vec {v_{n}}}} полна, тогда и только тогда, когда матрица A не вырождена (det Aneq 0).

Доказательство

(1) Доказательство основано на том, что вектор {displaystyle sum x_{i}{vec {v_{i}}}} имеет столбец координат, равный {displaystyle AX}, где {displaystyle X=(x_{1},x_{2},dots ,x_{n})^{T}}.

Если {displaystyle sum x_{i}{vec {v_{i}}}=0}, то AX=0. Тогда {displaystyle (det A)cdot X=0} и если X отличен от нуля, то det A=0.

Обратно, если det A=0, существует ненулевой столбец X, такой что AX=0. Это означает, что {displaystyle sum x_{i}{vec {v_{i}}}=0}.

(2) Если матрица A не вырождена, она обратима. Пусть {vec  {u}} — произвольный вектор, {displaystyle Y=(y_{1},y_{2},dots ,y_{n})^{T}} — столбец его координат, {displaystyle X=A^{-1}Y=(x_{1},x_{2},dots ,x_{n})^{T}}. Тогда {displaystyle {vec {u}}=sum x_{i}{vec {v_{i}}}}. Таким образом, произвольный вектор разложим по системы векторов {displaystyle {vec {v_{1}}},{vec {v_{2}}},dots ,{vec {v_{n}}}}, что означает её полноту.

Обратно, пусть матрица A вырождена. Тогда существует ненулевая строка коэффициентов {displaystyle C=(c_{1},c_{2},dots ,c_{n})}, такая, что {displaystyle CA=0}. Это значит, что любой вектор {displaystyle {vec {u}}=sum x_{i}{vec {v_{i}}}}, разложимый по системе векторов {displaystyle {vec {v_{1}}},{vec {v_{2}}},dots ,{vec {v_{n}}}}, удовлетворяет ограничению {displaystyle c_{1}x_{1}+c_{2}x_{2}+dots +c_{n}x_{n}=0}. Если какой-то коэффициент c_{k} отличен от нуля, то базисный вектор {displaystyle {vec {e_{k}}}} не разложим по этой системе векторов, и значит, она не полна.

Следствие. В векторном пространстве V, обладающем конечным базисом из n векторов:

(1) всякая система, состоящая менее, чем из n векторов, не полна;
(2) всякая система, состоящая более, чем из n векторов, линейно зависима;
(3) всякий базис пространства V содержит ровно n векторов.

Таким образом, размерность векторного пространства V с конечным базисом корректно определена.

Некоторые специальные свойства определителей[править | править код]

  • Определитель матрицы равен произведению её собственных значений.
  • Если квадратная матрица выражает линейное преобразование, то её определитель не меняется при замене базиса линейного пространства.

Алгоритмическая реализация[править | править код]

  • Прямые методы вычисления определителя могут быть основаны непосредственно на его определении, как суммы по перестановкам, или на разложении Лапласа по определителям меньшего порядка. Однако такие методы очень неэффективны, так как требуют О(n!) операций для вычисления определителя n-го порядка. В то же время они универсальны, применимы в тех случаях, когда элементы матрицы не являются числами (функции, многочлены, дифференциальные формы чётной степени и т. д.), и не требуют выполнения операций деления.
  • Можно вычислить определитель и выполнив меньшее количество операций умножения; точнее, можно для матрицы ntimes n обойтись лишь количеством слагаемых, равным количеству разбиений числа n вместо n!, например 5 вместо 6 для 3times3[6]:
    {displaystyle {begin{vmatrix}a_{11}&a_{12}&a_{13}\a_{21}&a_{22}&a_{23}\a_{31}&a_{32}&a_{33}end{vmatrix}}=(a_{11}+a_{12})(a_{21}+a_{23})(a_{31}+a_{32})-a_{12}a_{21}(a_{31}+a_{32}+a_{33})-a_{13}(a_{21}+a_{22}+a_{23})a_{31}-(a_{11}+a_{12}+a_{13})a_{23}a_{32}+a_{11}a_{22}a_{33}}.
  • Один из наиболее быстрых численных методов вычисления определителя заключается в простой модификации метода Гаусса. Следуя методу Гаусса, произвольную матрицу A можно привести к ступенчатому виду (Верхнетреугольная матрица), используя лишь две следующие операции над матрицей — перестановку двух строк и добавление к одной из строк матрицы другой строки, умноженной на произвольное число. Из свойств определителя следует, что вторая операция не изменяет определителя матрицы, а первая лишь меняет его знак на противоположный. Определитель матрицы, приведённой к ступенчатому виду, равен произведению элементов на её диагонали, так как она является треугольной, поэтому определитель исходной матрицы равен:
    {displaystyle det A=(-1)^{s}cdot det A_{mbox{ref}},}
где s — число перестановок строк, выполненных алгоритмом, а A_{mbox{ref}} — ступенчатая форма матрицы A, полученная в результате работы алгоритма. Сложность этого метода, как и метода Гаусса, составляет O(n^{3}), для его реализации необходимо использование операции деления.
  • Определитель можно вычислить, зная LU-разложение матрицы. Если A=LU, где L и U — треугольные матрицы, то det A=(det L)(det U). Определитель треугольной матрицы равен просто произведению её диагональных элементов.
  • Если доступен алгоритм, выполняющий умножение двух матриц порядка n за время M(n), где M(n)geqslant n^{a}, для некоторого a>2, то определитель матрицы порядка n может быть вычислен за время O(M(n)).[7] В частности это означает, что, используя для умножения матриц алгоритм Копперсмита — Винограда, определитель можно вычислить за время O(n^{2.376}).

Специальные виды определителей[править | править код]

  • Определитель Вронского (Вронскиан)
  • Определитель Вандермонда
  • Определитель Грама
  • Определитель Якоби (Якобиан)

См. также[править | править код]

  • Циркулянт
  • Перманент
  • Пфаффиан
  • Результант

Примечания[править | править код]

  1. Бронштейн И. Н., Семендяев К. А. Справочник по математике для инженеров и учащихся втузов. — 13-е изд., исправленное. — М.: Наука, 1986.
  2. Э. И. Березкина. Математика древнего Китая. — М.: Наука, 1980.
  3. H. W. Eves. An Introduction to the History of Mathematics. — Saunders College Publishing, 1990.
  4. Скорняков Л. А. Элементы алгебры. — М.: Наука, 1986. — С. 16-23. — Тираж 21 000 экз.
  5. Определитель матрицы и геометрический смысл определителя матрицы (рус.). Математика для всех.
  6. Houston, Robin; Goucher, Adam P. & Johnston, Nathaniel (2023), A New Formula for the Determinant and Bounds on Its Tensor and Waring Ranks, arΧiv:2301.06586 [math.CO].
  7. J. R. Bunch and J.E. Hopcroft. Triangular factorization and inversion by fast matrix multiplication, Mathematics of Computation, 28 (1974) 231—236.

Литература[править | править код]

  • В. А. Ильин, Э. Г. Позняк Линейная алгебра, М.: Наука — Физматлит, 1999.
  • Беклемишев Д. В. Курс аналитической геометрии и линейной алгебры. М.: Физматлит, 2000.
  • Кострикин А. И. Введение в алгебру. Часть 1. Основы алгебры: Учебник для вузов. М.: Физматлит, 2004.
  • Боревич З. И. Определители и матрицы. – М.: Наука, 1988.

Определитель матрицы и его свойства

8 февраля 2018

В этом уроке мы детально рассмотрим несколько ключевые вопросов и определений, благодаря чему вы раз и навсегда разберётесь и с матрицами, и с определителями, и со всеми их свойствами.

Определители — центральное понятие в алгебре матриц. Подобно формулам сокращённого умножения, они будут преследовать вас на протяжении всего курса высшей математики. Поэтому читаем, смотрим и разбираемся досконально.:)

И начнём мы с самого сокровенного — а что такое матрица? И как правильно с ней работать.

Правильная расстановка индексов в матрице

Матрица — это просто таблица, заполненная числами. Нео тут ни при чём.

Одна из ключевых характеристик матрицы — это её размерность, т.е. количество строк и столбцов, из которых она состоит. Обычно говорят, что некая матрица $A$ имеет размер $left[ mtimes n right]$, если в ней имеется $m$ строк и $n$ столбцов. Записывают это так:

[A=left[ mtimes n right]]

Или вот так:

[A=left( {{a}_{ij}} right),quad 1le ile m;quad 1le jle n.]

Бывают и другие обозначения — тут всё зависит от предпочтений лектора/ семинариста/ автора учебника. Но в любом случае со всеми этими $left[ mtimes n right]$ и ${{a}_{ij}}$ возникает одна и та же проблема:

Какой индекс за что отвечает? Сначала идёт номер строки, затем — столбца? Или наоборот?

При чтении лекций и учебников ответ будет казаться очевидным. Но когда на экзамене перед вами — только листик с задачей, можно переволноваться и внезапно запутаться.

Поэтому давайте разберёмся с этим вопросом раз и навсегда. Для начала вспомним обычную систему координат из школьного курса математики:

Введение системы координат на плоскости

Помните её? У неё есть начало координат (точка $O=left( 0;0 right)$) оси $x$и $y$, а каждая точка на плоскости однозначно определяется по координатам: $A=left( 1;2 right)$, $B=left( 3;1 right)$ и т.д.

А теперь давайте возьмём эту конструкцию и поставим её рядом с матрицей так, чтобы начало координат находилось в левом верхнем углу. Почему именно там? Да потому что открывая книгу, мы начинаем читать именно с левого верхнего угла страницы — запомнить это легче лёгкого.

Но куда направить оси? Мы направим их так, чтобы вся наша виртуальная «страница» была охвачена этими осями. Правда, для этого придётся повернуть нашу систему координат. Единственно возможный вариант такого расположения:

Наложение системы координат на матрицу

Теперь всякая клетка матрицы имеет однозначные координаты $x$ и $y$. Например запись ${{a}_{24}}$ означает, что мы обращаемся к элементу с координатами $x=2$ и $y=4$. Размеры матрицы тоже однозначно задаются парой чисел:

Определение индексов в матрице

Просто всмотритесь в эту картинку внимательно. Поиграйтесь с координатами (особенно когда будете работать с настоящими матрицами и определителями) — и очень скоро поймёте, что даже в самых сложных теоремах и определениях вы прекрасно понимаете, о чём идёт речь.

Разобрались? Что ж, переходим к первому шагу просветления — геометрическому определению определителя.:)

Геометрическое определение

Прежде всего хотел бы отметить, что определитель существует только для квадратных матриц вида $left[ ntimes n right]$. Определитель — это число, которое cчитается по определённым правилам и является одной из характеристик этой матрицы (есть другие характеристики: ранг, собственные вектора, но об этом в других уроках).

Ну и что это за характеристика? Что он означает? Всё просто:

Определитель квадратной матрицы $A=left[ ntimes n right]$ — это объём $n$-мерного параллелепипеда, который образуется, если рассмотреть строки матрицы в качестве векторов, образующих рёбра этого параллелепипеда.

Например, определитель матрицы размера 2×2 — это просто площадь параллелограмма, а для матрицы 3×3 это уже объём 3-мерного параллелепипеда — того самого, который так бесит всех старшеклассников на уроках стереометрии.

На первый взгляд это определение может показаться совершенно неадекватным. Но давайте не будем спешить с выводами — глянем на примеры. На самом деле всё элементарно, Ватсон:

Задача. Найдите определители матриц:

[left| begin{matrix} 1 & 0 \ 0 & 3 \end{matrix} right|quad left| begin{matrix} 1 & -1 \ 2 & 2 \end{matrix} right|quad left| begin{matrix}2 & 0 & 0 \ 1 & 3 & 0 \ 1 & 1 & 4 \end{matrix} right|]

Решение. Первые два определителя имеют размер 2×2. Значит, это просто площади параллелограммов. Начертим их и посчитаем площадь.

Первый параллелограмм построен на векторах ${{v}_{1}}=left( 1;0 right)$ и ${{v}_{2}}=left( 0;3 right)$:

Определитель 2×2 — это площадь параллелограмма

Очевидно, это не просто параллелограмм, а вполне себе прямоугольник. Его площадь равна

[S=1cdot 3=3]

Второй параллелограмм построен на векторах ${{v}_{1}}=left( 1;-1 right)$ и ${{v}_{2}}=left( 2;2 right)$. Ну и что с того? Это тоже прямоугольник:

Ещё один определитель 2×2

Стороны этого прямоугольника (по сути — длины векторов) легко считаются по теореме Пифагора:

[begin{align} & left| {{v}_{1}} right|=sqrt{{{1}^{2}}+{{left( -1 right)}^{2}}}=sqrt{2}; \ & left| {{v}_{2}} right|=sqrt{{{2}^{2}}+{{2}^{2}}}=sqrt{8}=2sqrt{2}; \ & S=left| {{v}_{1}} right|cdot left| {{v}_{2}} right|=sqrt{2}cdot 2sqrt{2}=4. \end{align}]

Осталось разобраться с последним определителем — там уже матрица 3×3. Придётся вспоминать стереометрию:

Определитель 3×3 — это объём параллелепипеда

Выглядит мозговыносяще, но по факту достаточно вспомнить формулу объёма параллелепипеда:

[V=Scdot h]

где $S$ — площадь основания (в нашем случае это площадь параллелограмма на плоскости $OXY$), $h$ — высота, проведённая к этому основанию (по сути, $z$-координата вектора ${{v}_{3}}$).

Площадь параллелограмма (мы начертили его отдельно) тоже считается легко:

[begin{align} & S=2cdot 3=6; \ & V=Scdot h=6cdot 4=24. \end{align}]

Вот и всё! Записываем ответы.

Ответ: 3; 4; 24.

Небольшое замечание по поводу системы обозначений. Кому-то наверняка не понравится, что я игнорирую «стрелочки» над векторами. Якобы так можно спутать вектор с точкой или ещё с чем.

Но давайте серьёзно: мы с вами уже взрослые мальчики и девочки, поэтому из контекста прекрасно понимаем, когда речь идёт о векторе, а когда — о точке. Стрелки лишь засоряют повествование, и без того под завязку напичканное математическими формулами.

И ещё. В принципе, ничто не мешает рассмотреть и определитель матрицы 1×1 — такая матрица представляет собой просто одну клетку, а число, записанное в этой клетке, и будет определителем. Но тут есть важное замечание:

В отличие от классического объёма, определитель даст нам так называемый «ориентированный объём», т.е. объём с учётом последовательности рассмотрения векторов-строк.

И если вы хотите получить объём в классическом смысле этого слова, придётся взять модуль определителя, но сейчас не стоит париться об этом — всё равно через несколько секунд мы научимся считать любой определитель с любыми знаками, размерами и т.д.:)

Алгебраическое определение

При всей красоте и наглядности геометрического подхода у него есть серьёзный недостаток: он ничего не говорит нам о том, как этот самый определитель считать.

Поэтому сейчас мы разберём альтернативное определение — алгебраическое. Для этого нам потребуется краткая теоретическая подготовка, зато на выходе мы получим инструмент, позволяющий считать в матрицах что и как угодно.

Правда, там появится новая проблема… но обо всём по порядку.

Перестановки и инверсии

Давайте выпишем в строчку числа от 1 до $n$. Получится что-то типа этого:

[1;2;3;4;5;…;n-1;n]

Теперь (чисто по приколу) поменяем парочку чисел местами. Можно поменять соседние:

[1;3;2;4;5;…;n-1;n]

А можно — не особо соседние:

[n;2;3;4;5;…;n-1;1]

И знаете, что? А ничего! В алгебре эта хрень называется перестановкой. И у неё есть куча свойств.

Определение. Перестановка длины $n$ — строка из $n$ различных чисел, записанных в любой последовательности. Обычно рассматриваются первые $n$ натуральных чисел (т.е. как раз числа 1, 2, …, $n$), а затем их перемешивают для получения нужной перестановки.

Обозначаются перестановки так же, как и векторы — просто буквой и последовательным перечислением своих элементов в скобках. Например: $p=left( 1;3;2 right)$ или $p=left( 2;5;1;4;3 right)$. Буква может быть любой, но пусть будет $p$.:)

Далее для простоты изложения будем работать с перестановками длины 5 — они уже достаточно серьёзны для наблюдения всяких подозрительных эффектов, но ещё не настолько суровы для неокрепшего мозга, как перестановки длины 6 и более. Вот примеры таких перестановок:

[begin{align} & {{p}_{1}}=left( 1;2;3;4;5 right) \ & {{p}_{2}}=left( 1;3;2;5;4 right) \ & {{p}_{3}}=left( 5;4;3;2;1 right) \end{align}]

Естественно, перестановку длины $n$ можно рассматривать как функцию, которая определена на множестве $left{ 1;2;…;n right}$ и биективно отображает это множество на себя же. Возвращаясь к только что записанным перестановкам ${{p}_{1}}$, ${{p}_{2}}$ и ${{p}_{3}}$, мы вполне законно можем написать:

[{{p}_{1}}left( 1 right)=1;{{p}_{2}}left( 3 right)=2;{{p}_{3}}left( 2 right)=4;]

Количество различных перестановок длины $n$ всегда ограничено и равно $n!$ — это легко доказуемый факт из комбинаторики. Например, если мы захотим выписать все перестановки длины 5, то мы весьма заколебёмся, поскольку таких перестановок будет

[n!=5!=1cdot 2cdot 3cdot 4cdot 5=120]

Одной из ключевых характеристик всякой перестановки является количество инверсий в ней.

Определение. Инверсия в перестановке $p=left( {{a}_{1}};{{a}_{2}};…;{{a}_{n}} right)$ — всякая пара $left( {{a}_{i}};{{a}_{j}} right)$ такая, что $i lt j$, но ${{a}_{i}} gt {{a}_{j}}$. Проще говоря, инверсия — это когда большее число стоит левее меньшего (не обязательно соседнего).

Мы будем обозначать через $Nleft( p right)$ количество инверсий в перестановке $p$, но будьте готовы встретиться и с другими обозначениями в разных учебниках и у разных авторов — единых стандартов тут нет. Тема инверсий весьма обширна, и ей будет посвящён отдельный урок. Сейчас же наша задача — просто научиться считать их в реальных задачах.

Например, посчитаем количество инверсий в перестановке $p=left( 1;4;5;3;2 right)$:

[left( 4;3 right);left( 4;2 right);left( 5;3 right);left( 5;2 right);left( 3;2 right).]

Таким образом, $Nleft( p right)=5$. Как видите, ничего страшного в этом нет. Сразу скажу: дальше нас будет интересовать не столько само число $Nleft( p right)$, сколько его чётность/ нечётность. И тут мы плавно переходим к ключевому термину сегодняшнего урока.

Что такое определитель

Пусть дана квадратная матрица $A=left[ ntimes n right]$. Тогда:

Определение. Определитель матрицы $A=left[ ntimes n right]$ — это алгебраическая сумма $n!$ слагаемых, составленных следующим образом. Каждое слагаемое — это произведение $n$ элементов матрицы, взятых по одному из каждой строки и каждого столбца, умноженное на (−1) в степени количество инверсий:

[left| A right|=sumlimits_{n!}{{{left( -1 right)}^{Nleft( p right)}}cdot {{a}_{1;pleft( 1 right)}}cdot {{a}_{2;pleft( 2 right)}}cdot …cdot {{a}_{n;pleft( n right)}}}]

Принципиальным моментом при выборе множителей для каждого слагаемого в определителе является тот факт, что никакие два множителя не стоят в одной строчке или в одном столбце.

Благодаря этому можно без ограничения общности считать, что индексы $i$ множителей ${{a}_{i;j}}$ «пробегают» значения 1, …, $n$, а индексы $j$ являются некоторой перестановкой от первых:

[j=pleft( i right),quad i=1,2,…,n]

А когда есть перестановка $p$, мы легко посчитаем инверсии $Nleft( p right)$ — и очередное слагаемое определителя готово.

Естественно, никто не запрещает поменять местами множители в каком-либо слагаемом (или во всех сразу — чего мелочиться-то?), и тогда первые индексы тоже будут представлять собой некоторую перестановку. Но в итоге ничего не поменяется: суммарное количество инверсий в индексах $i$ и $j$ сохраняет чётность при подобных извращениях, что вполне соответствует старому-доброму правилу:

От перестановки множителей произведение чисел не меняется.

Вот только не надо приплетать это правило к умножению матриц — в отличие от умножения чисел, оно не коммутативно. Но это я отвлёкся.:)

Матрица 2×2

Вообще-то можно рассмотреть и матрицу 1×1 — это будет одна клетка, и её определитель, как нетрудно догадаться, равен числу, записанному в этой клетке. Ничего интересного.

Поэтому давайте рассмотрим квадратную матрицу размером 2×2:

[left[ begin{matrix} {{a}_{11}} & {{a}_{12}} \ {{a}_{21}} & {{a}_{22}} \end{matrix} right]]

Поскольку количество строк в ней $n=2$, то определитель будет содержать $n!=2!=1cdot 2=2$ слагаемых. Выпишем их:

[begin{align} & {{left( -1 right)}^{Nleft( 1;2 right)}}cdot {{a}_{11}}cdot {{a}_{22}}={{left( -1 right)}^{0}}cdot {{a}_{11}}cdot {{a}_{22}}={{a}_{11}}{{a}_{22}}; \ & {{left( -1 right)}^{Nleft( 2;1 right)}}cdot {{a}_{12}}cdot {{a}_{21}}={{left( -1 right)}^{1}}cdot {{a}_{12}}cdot {{a}_{21}}={{a}_{12}}{{a}_{21}}. \end{align}]

Очевидно, что в перестановке $left( 1;2 right)$, состоящей из двух элементов, нет инверсий, поэтому $Nleft( 1;2 right)=0$. А вот в перестановке $left( 2;1 right)$ одна инверсия имеется (собственно, 2 < 1), поэтому $Nleft( 2;1 right)=1.$

Итого универсальная формула вычисления определителя для матрицы 2×2 выглядит так:

[left| begin{matrix} {{a}_{11}} & {{a}_{12}} \ {{a}_{21}} & {{a}_{22}} \end{matrix} right|={{a}_{11}}{{a}_{22}}-{{a}_{12}}{{a}_{21}}]

Графически это можно представить как произведение элементов, стоящих на главной диагонали, минус произведение элементов на побочной:

Определитель матрицы 2×2

Рассмотрим пару примеров:

Задача. Вычислите определитель:

[left| begin{matrix} 5 & 6 \ 8 & 9 \end{matrix} right|;quad left| begin{matrix} 7 & 12 \ 14 & 1 \end{matrix} right|.]

Решение. Всё считается в одну строчку. Первая матрица:

[5cdot 9-8cdot 6=45-48=-3]

И вторая:

[7cdot 1-14cdot 12=7-168=-161]

Ответ: −3; −161.

Впрочем, это было слишком просто. Давайте рассмотрим матрицы 3×3 — там уже интересно.

Матрица 3×3

Теперь рассмотрим квадратную матрицу размера 3×3:

[left[ begin{matrix} {{a}_{11}} & {{a}_{12}} & {{a}_{13}} \ {{a}_{21}} & {{a}_{22}} & {{a}_{23}} \ {{a}_{31}} & {{a}_{32}} & {{a}_{33}} \end{matrix} right]]

При вычислении её определителя мы получим $3!=1cdot 2cdot 3=6$ слагаемых — ещё не слишком много для паники, но уже достаточно, чтобы начать искать какие-то закономерности. Для начала выпишем все перестановки из трёх элементов и посчитаем инверсии в каждой из них:

[begin{align} & {{p}_{1}}=left( 1;2;3 right)Rightarrow Nleft( {{p}_{1}} right)=Nleft( 1;2;3 right)=0; \ & {{p}_{2}}=left( 1;3;2 right)Rightarrow Nleft( {{p}_{2}} right)=Nleft( 1;3;2 right)=1; \ & {{p}_{3}}=left( 2;1;3 right)Rightarrow Nleft( {{p}_{3}} right)=Nleft( 2;1;3 right)=1; \ & {{p}_{4}}=left( 2;3;1 right)Rightarrow Nleft( {{p}_{4}} right)=Nleft( 2;3;1 right)=2; \ & {{p}_{5}}=left( 3;1;2 right)Rightarrow Nleft( {{p}_{5}} right)=Nleft( 3;1;2 right)=2; \ & {{p}_{6}}=left( 3;2;1 right)Rightarrow Nleft( {{p}_{6}} right)=Nleft( 3;2;1 right)=3. \end{align}]

Как и предполагалось, всего выписано 6 перестановок ${{p}_{1}}$, … ${{p}_{6}}$ (естественно, можно было бы выписать их в другой последовательности — суть от этого не изменится), а количество инверсий в них меняется от 0 до 3.

В общем, у нас будет три слагаемых с «плюсом» (там, где $Nleft( p right)$ — чётное) и ещё три с «минусом». А в целом определитель будет считаться по формуле:

[left| begin{matrix} {{a}_{11}} & {{a}_{12}} & {{a}_{13}} \ {{a}_{21}} & {{a}_{22}} & {{a}_{23}} \ {{a}_{31}} & {{a}_{32}} & {{a}_{33}} \end{matrix} right|=begin{matrix} {{a}_{11}}{{a}_{22}}{{a}_{33}}+{{a}_{12}}{{a}_{23}}{{a}_{31}}+{{a}_{13}}{{a}_{21}}{{a}_{32}}- \ -{{a}_{13}}{{a}_{22}}{{a}_{31}}-{{a}_{12}}{{a}_{21}}{{a}_{33}}-{{a}_{11}}{{a}_{23}}{{a}_{32}} \end{matrix}]

Вот только не надо сейчас садиться и яростно зубрить все эти индексы! Вместо непонятных цифр лучше запомните следующее мнемоническое правило:

Правило треугольника. Для нахождения определителя матрицы 3×3 нужно сложить три произведения элементов, стоящих на главной диагонали и в вершинах равнобедренных треугольников со стороной, параллельной этой диагонали, а затем вычесть такие же три произведения, но на побочной диагонали. Схематически это выглядит так:

Определитель матрицы 3×3: правило треугольников

Именно эти треугольники (или пентаграммы — кому как больше нравится) любят рисовать во всяких учебниках и методичках по алгебре. Впрочем, не будем о грустном. Давайте лучше посчитаем один такой определитель — для разминки перед настоящей жестью.:)

Задача. Вычислите определитель:

[left| begin{matrix} 1 & 2 & 3 \ 4 & 5 & 6 \ 7 & 8 & 1 \end{matrix} right|]

Решение. Работаем по правилу треугольников. Сначала посчитаем три слагаемых, составленных из элементов на главной диагонали и параллельно ей:

[begin{align} & 1cdot 5cdot 1+2cdot 6cdot 7+3cdot 4cdot 8= \ & =5+84+96=185 \end{align}]

Теперь разбираемся с побочной диагональю:

[begin{align} & 3cdot 5cdot 7+2cdot 4cdot 1+1cdot 6cdot 8= \ & =105+8+48=161 \end{align}]

Осталось лишь вычесть из первого числа второе — и мы получим ответ:

[185-161=24]

Вот и всё!

Ответ: 24.

Тем не менее, определители матриц 3×3 — это ещё не вершина мастерства. Самое интересное ждёт нас дальше.:)

Общая схема вычисления определителей

Как мы знаем, с ростом размерности матрицы $n$ количество слагаемых в определителе составляет $n!$ и быстро растёт. Всё-таки факториал — это вам не хрен собачий довольно быстро растущая функция.

Уже для матриц 4×4 считать определители напролом (т.е. через перестановки) становится как-то не оч. Про 5×5 и более вообще молчу. Поэтому к делу подключаются некоторые свойства определителя, но для их понимания нужна небольшая теоретическая подготовка.

Готовы? Поехали!

Что такое минор матрицы

Пусть дана произвольная матрица $A=left[ mtimes n right]$. Заметьте: не обязательно квадратная. В отличие от определителей, миноры — это такие няшки, которые существуют не только в суровых квадратных матрицах. Выберем в этой матрице несколько (например, $k$) строк и столбцов, причём $1le kle m$ и $1le kle n$. Тогда:

Определение. Минор порядка $k$ — определитель квадратной матрицы, возникающей на пересечении выбранных $k$ столбцов и строк. Также минором мы будем называть и саму эту новую матрицу.

Обозначается такой минор ${{M}_{k}}$. Естественно, у одной матрицы может быть целая куча миноров порядка $k$. Вот пример минора порядка 2 для матрицы $left[ 5times 6 right]$:

Выбор $k = 2$ столбцов и строк для формирования минора

Совершенно необязательно, чтобы выбранные строки и столбцы стояли рядом, как в рассмотренном примере. Главное, чтобы количество выбранных строк и столбцов было одинаковым (это и есть число $k$).

Есть и другое определение. Возможно, кому-то оно больше придётся по душе:

Определение. Пусть дана прямоугольная матрица $A=left[ mtimes n right]$. Если после вычеркивания в ней одного или нескольких столбцов и одной или нескольких строк образуется квадратная матрица размера $left[ ktimes k right]$, то её определитель — это и есть минор ${{M}_{k}}$. Саму матрицу мы тоже иногда будем называть минором — это будет ясно из контекста.

Как говорил мой кот, иногда лучше один раз навернуться с 11-го этажа есть корм, чем мяукать, сидя на балконе.

Пример. Пусть дана матрица

[A=left[ begin{matrix} begin{matrix} 1 \ 2 \ 3 \end{matrix} & begin{matrix} 7 \ 4 \ 0 \end{matrix} & begin{matrix} 9 \ 5 \ 6 \end{matrix} & begin{matrix} 0 \ 3 \ 1 \end{matrix} \end{matrix} right]]

Выбирая строку 1 и столбец 2, получаем минор первого порядка:

[{{M}_{1}}=left| 7 right|=7]

Выбирая строки 2, 3 и столбцы 3, 4, получаем минор второго порядка:

[{{M}_{2}}=left| begin{matrix} 5 & 3 \ 6 & 1 \end{matrix} right|=5-18=-13]

А если выбрать все три строки, а также столбцы 1, 2, 4, будет минор третьего порядка:

[{{M}_{3}}=left| begin{matrix} 1 & 7 & 0 \ 2 & 4 & 3 \ 3 & 0 & 1 \end{matrix} right|]

Считать этот определитель мне уже в лом. Но он равен 53.:)

Читателю не составит труда найти и другие миноры порядков 1, 2 или 3. Поэтому идём дальше.

Алгебраические дополнения

«Ну ok, и что дают нам эти миньоны миноры?» — наверняка спросите вы. Сами по себе — ничего. Но в квадратных матрицах у каждого минора появляется «компаньон» — дополнительный минор, а также алгебраическое дополнение. И вместе эти два ушлёпка позволят нам щёлкать определители как орешки.

Определение. Пусть дана квадратная матрица $A=left[ ntimes n right]$, в которой выбран минор ${{M}_{k}}$. Тогда дополнительный минор для минора ${{M}_{k}}$ — это кусок исходной матрицы $A$, который останется при вычёркивании всех строк и столбцов, задействованных при составлении минора ${{M}_{k}}$:

Дополнительный минор к минору ${{M}_{2}}$

Уточним один момент: дополнительный минор — это не просто «кусок матрицы», а определитель этого куска.

Обозначаются дополнительные миноры с помощью «звёздочки»: $M_{k}^{*}$:

[M_{k}^{*}=left| Anabla {{M}_{k}} right|]

где операция $Anabla {{M}_{k}}$ буквально означает «вычеркнуть из $A$ строки и столбцы, входящие в ${{M}_{k}}$». Эта операция не является общепринятой в математике — я её сам только что придумал для красоты повествования.:)

Дополнительные миноры редко используются сами по себе. Они являются частью более сложной конструкции — алгебраического дополнения.

Определение. Алгебраическое дополнение минора ${{M}_{k}}$ — это дополнительный минор $M_{k}^{*}$, умноженный на величину ${{left( -1 right)}^{S}}$, где $S$ — сумма номеров всех строк и столбцов, задействованных в исходном миноре ${{M}_{k}}$.

Как правило, алгебраическое дополнение минора ${{M}_{k}}$ обозначается через ${{A}_{k}}$. Поэтому:

[{{A}_{k}}={{left( -1 right)}^{S}}cdot M_{k}^{*}]

Сложно? На первый взгляд — да. Но это не точно. Потому что на самом деле всё легко. Рассмотрим пример:

Пример. Дана матрица 4×4:

[A=left[ begin{matrix} 1 & 2 & 3 & 4 \ 5 & 6 & 7 & 8 \ 9 & 10 & 11 & 12 \ 13 & 14 & 15 & 16 \end{matrix} right]]

Выберем минор второго порядка

[{{M}_{2}}=left| begin{matrix} 3 & 4 \ 15 & 16 \end{matrix} right|]

Капитан Очевидность как бы намекает нам, что при составлении этого минора были задействованы строки 1 и 4, а также столбцы 3 и 4. Вычёркиваем их — получим дополнительный минор:

[M_{2}^{*}=left| begin{matrix} 5 & 6 \ 9 & 10 \end{matrix} right|=50-54=-4]

Осталось найти число $S$ и получить алгебраическое дополнение. Поскольку мы знаем номера задействованных строк (1 и 4) и столбцов (3 и 4), всё просто:

[begin{align} & S=1+4+3+4=12; \ & {{A}_{2}}={{left( -1 right)}^{S}}cdot M_{2}^{*}={{left( -1 right)}^{12}}cdot left( -4 right)=-4end{align}]

Ответ: ${{A}_{2}}=-4$

Вот и всё! По сути, всё различие между дополнительным минором и алгебраическим дополнением — только в минусе спереди, да и то не всегда.

Наша задача сейчас — научиться быстро считать алгебраические дополнения, потому что они являются составной частью «Теоремы, Которую Нельзя Называть». Но мы всё же назовём. Встречайте:

Теорема Лапласа

И вот мы пришли к тому, зачем, собственно, все эти миноры и алгебраические дополнения были нужны.

Теорема Лапласа о разложении определителя. Пусть в матрице размера $left[ ntimes n right]$ выбрано $k$ строк (столбцов), причём $1le kle n-1$. Тогда определитель этой матрицы равен сумме всех произведений миноров порядка $k$, содержащихся в выбранных строках (столбцах), на их алгебраические дополнения:

[left| A right|=sum{{{M}_{k}}cdot {{A}_{k}}}]

Причём таких слагаемых будет ровно $C_{n}^{k}$.

Ладно, ладно: про $C_{n}^{k}$ — это я уже понтуюсь, в оригинальной теореме Лапласа ничего такого не было. Но комбинаторику никто не отменял, и буквально беглый взгляд на условие позволит вам самостоятельно убедиться, что слагаемых будет именно столько.:)

Мы не будем её доказывать, хоть это и не представляет особой трудности — все выкладки сводятся к старым-добрым перестановкам и чётности/ нечётности инверсий. Тем не менее, доказательство будет представлено в отдельном параграфе, а сегодня у нас сугубо практический урок.

Поэтому переходим к частному случаю этой теоремы, когда миноры представляют собой отдельные клетки матрицы.

Разложение определителя по строке и столбцу

То, о чём сейчас пойдёт речь — как раз и есть основной инструмент работы с определителями, ради которого затевались вся эта дичь с перестановками, минорами и алгебраическими дополнениями.

Читайте и наслаждайтесь:

Следствие из Теоремы Лапласа (разложение определителя по строке/столбцу). Пусть в матрице размера $left[ ntimes n right]$ выбрана одна строка. Минорами в этой строке будут $n$ отдельных клеток:

[{{M}_{1}}={{a}_{ij}},quad j=1,…,n]

Дополнительные миноры тоже легко считаются: просто берём исходную матрицу и вычёркиваем строку и столбец, содержащие ${{a}_{ij}}$. Назовём такие миноры $M_{ij}^{*}$.

Для алгебраического дополнения ещё нужно число $S$, но в случае с минором порядка 1 это просто сумма «координат» клетки ${{a}_{ij}}$:

[S=i+j]

И тогда исходный определитель можно расписать через ${{a}_{ij}}$ и $M_{ij}^{*}$ согласно теореме Лапласа:

[left| A right|=sumlimits_{j=1}^{n}{{{a}_{ij}}cdot {{left( -1 right)}^{i+j}}cdot {{M}_{ij}}}]

Это и есть формула разложения определителя по строке. Но то же верно и для столбцов.

Из этого следствия можно сразу сформулировать несколько выводов:

  1. Эта схема одинаково хорошо работает как для строк, так и для столбцов. На самом деле чаще всего разложение будет идти именно по столбцам, нежели по строкам.
  2. Количество слагаемых в разложении всегда ровно $n$. Это существенно меньше $C_{n}^{k}$ и уж тем более $n!$.
  3. Вместо одного определителя $left[ ntimes n right]$ придётся считать несколько определителей размера на единицу меньше: $left[ left( n-1 right)times left( n-1 right) right]$.

Последний факт особенно важен. Например, вместо зверского определителя 4×4 теперь достаточно будет посчитать несколько определителей 3×3 — с ними мы уж как-нибудь справимся.:)

Что ж, попробуем посчитать одну такую задачку?

Задача. Найдите определитель:

[left| begin{matrix} 1 & 2 & 3 \ 4 & 5 & 6 \ 7 & 8 & 9 \end{matrix} right|]

Решение. Разложим этот определитель по первой строке:

[begin{align} left| A right|=1cdot {{left( -1 right)}^{1+1}}cdot left| begin{matrix} 5 & 6 \ 8 & 9 \end{matrix} right|+ & \ 2cdot {{left( -1 right)}^{1+2}}cdot left| begin{matrix} 4 & 6 \ 7 & 9 \end{matrix} right|+ & \ 3cdot {{left( -1 right)}^{1+3}}cdot left| begin{matrix} 4 & 5 \ 7 & 8 \end{matrix} right|= & \end{align}]

[begin{align} & =1cdot left( 45-48 right)-2cdot left( 36-42 right)+3cdot left( 32-35 right)= \ & =1cdot left( -3 right)-2cdot left( -6 right)+3cdot left( -3 right)=0. \end{align}]

Ответ: 0.

Задача. Найдите определитель:

[left| begin{matrix} 0 & 1 & 1 & 0 \ 1 & 0 & 1 & 1 \ 1 & 1 & 0 & 1 \ 1 & 1 & 1 & 0 \end{matrix} right|]

Решение. Для разнообразия давайте в этот раз работать со столбцами. Например, в последнем столбце присутствуют сразу два нуля — очевидно, это значительно сократит вычисления. Сейчас увидите почему.

Итак, раскладываем определитель по четвёртому столбцу:

[begin{align} left| begin{matrix} 0 & 1 & 1 & 0 \ 1 & 0 & 1 & 1 \ 1 & 1 & 0 & 1 \ 1 & 1 & 1 & 0 \end{matrix} right|=0cdot {{left( -1 right)}^{1+4}}cdot left| begin{matrix} 1 & 0 & 1 \ 1 & 1 & 0 \ 1 & 1 & 1 \end{matrix} right|+ & \ +1cdot {{left( -1 right)}^{2+4}}cdot left| begin{matrix} 0 & 1 & 1 \ 1 & 1 & 0 \ 1 & 1 & 1 \end{matrix} right|+ & \ +1cdot {{left( -1 right)}^{3+4}}cdot left| begin{matrix} 0 & 1 & 1 \ 1 & 0 & 1 \ 1 & 1 & 1 \end{matrix} right|+ & \ +0cdot {{left( -1 right)}^{4+4}}cdot left| begin{matrix} 0 & 1 & 1 \ 1 & 0 & 1 \ 1 & 1 & 0 \end{matrix} right| & \end{align}]

И тут — о, чудо! — два слагаемых сразу улетают коту под хвост, поскольку в них есть множитель «0». Остаётся ещё два определителя 3×3, с которыми мы легко разберёмся:

[begin{align} & left| begin{matrix} 0 & 1 & 1 \ 1 & 1 & 0 \ 1 & 1 & 1 \end{matrix} right|=0+0+1-1-1-0=-1; \ & left| begin{matrix} 0 & 1 & 1 \ 1 & 0 & 1 \ 1 & 1 & 1 \end{matrix} right|=0+1+1-0-0-1=1. \end{align}]

Возвращаемся к исходнику и находим ответ:

[left| begin{matrix} 0 & 1 & 1 & 0 \ 1 & 0 & 1 & 1 \ 1 & 1 & 0 & 1 \ 1 & 1 & 1 & 0 \end{matrix} right|=1cdot left( -1 right)+left( -1 right)cdot 1=-2]

Ну вот и всё. И никаких 4! = 24 слагаемых считать не пришлось.:)

Ответ: −2

Основные свойства определителя

В последней задаче мы видели, как наличие нулей в строках (столбцах) матрицы резко упрощает разложение определителя и вообще все вычисления. Возникает естественный вопрос: а нельзя ли сделать так, чтобы эти нули появились даже в той матрице, где их изначально не было?

Ответ однозначен: можно. И здесь нам на помощь приходят свойства определителя:

  1. Если поменять две строчки (столбца) местами, определитель поменяет знак;
  2. Если одну строку (столбец) умножить на число $k$, то весь определитель тоже умножится на число $k$;
  3. Если взять одну строку и прибавить (вычесть) её сколько угодно раз из другой, определитель не изменится;
  4. Если две строки определителя одинаковы, либо пропорциональны, либо одна из строк заполнена нулями, то весь определитель равен нулю;
  5. Все указанные выше свойства верны и для столбцов.
  6. При транспонировании матрицы определитель не меняется;
  7. Определитель произведения матриц равен произведению определителей.

Особую ценность представляет третье свойство: мы можем вычитать из одной строки (столбца) другую до тех пор, пока в нужных местах не появятся нули.

Чаще всего расчёты сводится к тому, чтобы «обнулить» весь столбец везде, кроме одного элемента, а затем разложить определитель по этому столбцу, получив матрицу размером на 1 меньше.

Давайте посмотрим, как это работает на практике:

Задача. Найдите определитель:

[left| begin{matrix} 1 & 2 & 3 & 4 \ 4 & 1 & 2 & 3 \ 3 & 4 & 1 & 2 \ 2 & 3 & 4 & 1 \end{matrix} right|]

Решение. Нулей тут как бы вообще не наблюдается, поэтому можно «долбить» по любой строке или столбцу — объём вычислений будет примерно одинаковым. Давайте не будем мелочиться и «обнулим» первый столбец: в нём уже есть клетка с единицей, поэтому просто возьмём первую строчку и вычтем её 4 раза из второй, 3 раза из третьей и 2 раза из последней.

В результате мы получим новую матрицу, но её определитель будет тем же:

[begin{matrix} left| begin{matrix} 1 & 2 & 3 & 4 \ 4 & 1 & 2 & 3 \ 3 & 4 & 1 & 2 \ 2 & 3 & 4 & 1 \end{matrix} right|begin{matrix} downarrow \ -4 \ -3 \ -2 \end{matrix}= \ =left| begin{matrix} 1 & 2 & 3 & 4 \ 4-4cdot 1 & 1-4cdot 2 & 2-4cdot 3 & 3-4cdot 4 \ 3-3cdot 1 & 4-3cdot 2 & 1-3cdot 3 & 2-3cdot 4 \ 2-2cdot 1 & 3-2cdot 2 & 4-2cdot 3 & 1-2cdot 4 \end{matrix} right|= \ =left| begin{matrix} 1 & 2 & 3 & 4 \ 0 & -7 & -10 & -13 \ 0 & -2 & -8 & -10 \ 0 & -1 & -2 & -7 \end{matrix} right| \end{matrix}]

Теперь с невозмутимостью Пятачка раскладываем этот определитель по первому столбцу:

[begin{matrix} 1cdot {{left( -1 right)}^{1+1}}cdot left| begin{matrix} -7 & -10 & -13 \ -2 & -8 & -10 \ -1 & -2 & -7 \end{matrix} right|+0cdot {{left( -1 right)}^{2+1}}cdot left| … right|+ \ +0cdot {{left( -1 right)}^{3+1}}cdot left| … right|+0cdot {{left( -1 right)}^{4+1}}cdot left| … right| \end{matrix}]

Понятно, что «выживет» только первое слагаемое — в остальных я даже определители не выписывал, поскольку они всё равно умножаются на ноль. Коэффициент перед определителем равен единице, т.е. его можно не записывать.

Зато можно вынести «минусы» из всех трёх строк определителя. По сути, мы трижды вынесли множитель (−1):

[left| begin{matrix} -7 & -10 & -13 \ -2 & -8 & -10 \ -1 & -2 & -7 \end{matrix} right|=cdot left| begin{matrix} 7 & 10 & 13 \ 2 & 8 & 10 \ 1 & 2 & 7 \end{matrix} right|]

Получили мелкий определитель 3×3, который уже можно посчитать по правилу треугольников. Но мы попробуем разложить и его по первому столбцу — благо в последней строчке гордо стоит единица:

[begin{align} & left( -1 right)cdot left| begin{matrix} 7 & 10 & 13 \ 2 & 8 & 10 \ 1 & 2 & 7 \end{matrix} right|begin{matrix} -7 \ -2 \ uparrow \end{matrix}=left( -1 right)cdot left| begin{matrix} 0 & -4 & -36 \ 0 & 4 & -4 \ 1 & 2 & 7 \end{matrix} right|= \ & =cdot left| begin{matrix} -4 & -36 \ 4 & -4 \end{matrix} right|=left( -1 right)cdot left| begin{matrix} -4 & -36 \ 4 & -4 \end{matrix} right| \end{align}]

Можно, конечно, ещё поприкалываться и разложить матрицу 2×2 по строке (столбцу), но мы же с вами адекватны, поэтому просто посчитаем ответ:

[left( -1 right)cdot left| begin{matrix} -4 & -36 \ 4 & -4 \end{matrix} right|=left( -1 right)cdot left( 16+144 right)=-160]

Вот так и разбиваются мечты. Всего-то −160 в ответе.:)

Ответ: −160.

Парочка замечаний перед тем, как мы перейдём к последней задаче:

  1. Исходная матрица была симметрична относительно побочной диагонали. Все миноры в разложении тоже симметричны относительно той же побочной диагонали.
  2. Строго говоря, мы могли вообще ничего не раскладывать, а просто привести матрицу к верхнетреугольному виду, когда под главной диагональю стоят сплошные нули. Тогда (в точном соответствии с геометрической интерпретацией, кстати) определитель равен произведению ${{a}_{ii}}$ — чисел на главной диагонали.

Идём дальше. Последняя задача в сегодняшнем уроке.

Задача. Найдите определитель:

[left| begin{matrix} 1 & 1 & 1 & 1 \ 2 & 4 & 8 & 16 \ 3 & 9 & 27 & 81 \ 5 & 25 & 125 & 625 \end{matrix} right|]

Решение. Ну, тут первая строка прямо-таки напрашивается на «обнуление». Берём первый столбец и вычитаем ровно один раз из всех остальных:

[begin{align} & left| begin{matrix} 1 & 1 & 1 & 1 \ 2 & 4 & 8 & 16 \ 3 & 9 & 27 & 81 \ 5 & 25 & 125 & 625 \end{matrix} right|= \ & =left| begin{matrix} 1 & 1-1 & 1-1 & 1-1 \ 2 & 4-2 & 8-2 & 16-2 \ 3 & 9-3 & 27-3 & 81-3 \ 5 & 25-5 & 125-5 & 625-5 \end{matrix} right|= \ & =left| begin{matrix} 1 & 0 & 0 & 0 \ 2 & 2 & 6 & 14 \ 3 & 6 & 24 & 78 \ 5 & 20 & 120 & 620 \end{matrix} right| \end{align}]

Раскладываем по первой строке, а затем выносим общие множители из оставшихся строк:

[cdot left| begin{matrix} 2 & 6 & 14 \ 6 & 24 & 78 \ 20 & 120 & 620 \end{matrix} right|=cdot left| begin{matrix} 1 & 3 & 7 \ 1 & 4 & 13 \ 1 & 6 & 31 \end{matrix} right|]

Снова наблюдаем «красивые» числа, но уже в первом столбце — раскладываем определитель по нему:

[begin{align} & 240cdot left| begin{matrix} 1 & 3 & 7 \ 1 & 4 & 13 \ 1 & 6 & 31 \end{matrix} right|begin{matrix} downarrow \ -1 \ -1 \end{matrix}=240cdot left| begin{matrix} 1 & 3 & 7 \ 0 & 1 & 6 \ 0 & 3 & 24 \end{matrix} right|= \ & =240cdot {{left( -1 right)}^{1+1}}cdot left| begin{matrix} 1 & 6 \ 3 & 24 \end{matrix} right|= \ & =240cdot 1cdot left( 24-18 right)=1440 \end{align}]

Порядок. Задача решена.

Ответ: 1440

Всё. Хорош читать этот бред.:)

Смотрите также:

  1. Обратная матрица
  2. Умножение матриц
  3. Геометрическая вероятность
  4. Решение задач B12: №448—455
  5. Задачи на проценты: формула, упрощающая вычисления
  6. Задача B4 про три дороги — стандартная задача на движение

Мы уже знакомы с понятием определителя матрицы. Также мы рассмотрели правила вычисления детерминантов (определителей) первого и второго порядка. Познакомимся с различными вариантами нахождения определителей третьего порядка.

Вычисление определителей по правилу треугольника

Схематически раскрытие определителя по этому правилу выглядит так:

Как вычислить определитель матрицы третьего порядка 1.png

Согласно рисункам №1 и №2 мы перемножаем элементы, соединенные прямыми. Произведения элементов будут иметь определенные знаки: для рисунка 1 — «+», для рисунка 2 — «-».

Произведения, которые берутся со знаком «+» Произведения, которые берутся со знаком «-»
a11⋅a22⋅a33a_{11} cdot a_{22} cdot a_{33} a13⋅a22⋅a31a_{13}cdot a_{22}cdot a_{31}
a12⋅a23⋅a31a_{12} cdot a_{23} cdot a_{31} a12⋅a33⋅a21a_{12}cdot a_{33}cdot a_{21}
a13⋅a32⋅a21a_{13} cdot a_{32} cdot a_{21} a11⋅a23⋅a32a_{11}cdot a_{23}cdot a_{32}

На рисунке 1 мы видим равнобедренные треугольники с основаниями, параллельными главной диагонали; на рисунке 2 — равнобедренные треугольники с основаниями, параллельными второй (побочной) диагонали. Поэтому данное правило имеет такое название.

Определитель может быть вычислен по формуле:

∣a11a12a13a21a22a23a31a32a33∣=begin{vmatrix}a_{11}&a_{12}&a_{13}\a_{21}&a_{22}&a_{23}\a_{31}&a_{32}&a_{33}end{vmatrix}=

=a11⋅a22⋅a33+a12⋅a23⋅a31+a13⋅a32⋅a21−a13⋅a22⋅a31−a12⋅a33⋅a21−a11⋅a23⋅a32=a_{11}cdot a_{22}cdot a_{33}+a_{12}cdot a_{23}cdot a_{31}+a_{13}cdot a_{32}cdot a_{21}-a_{13}cdot a_{22}cdot a_{31}-a_{12}cdot a_{33}cdot a_{21}-a_{11}cdot a_{23}cdot a_{32}.

Примеры

Рассмотрим примеры нахождения определителя по правилу треугольника.

Пример 1

Найти определитель ∣925148637∣begin{vmatrix}9&2&5\1&4&8\6&3&7end{vmatrix} по правилу треугольника.

По правилу треугольника определитель третьего порядка равен:

∣925148637∣=9⋅4⋅7+2⋅8⋅6+5⋅3⋅1−5⋅4⋅6−2⋅7⋅1−9⋅8⋅3=begin{vmatrix}9&2&5\1&4&8\6&3&7end{vmatrix}=9cdot4cdot7+2cdot8cdot6+5cdot3cdot1-5cdot4cdot6-2cdot7cdot1-9cdot8cdot3=

=252+96+15−120−14−216=13=252+96+15-120-14-216=13.

Пример 2

Найти определитель ∣21−46−3510−1∣begin{vmatrix}2&1&-4\6&-3&5\1&0&-1end{vmatrix} по правилу треугольника.

Искомый определитель третьего порядка равен:

∣21−46−3510−1∣=begin{vmatrix}2&1&-4\6&-3&5\1&0&-1end{vmatrix}=

=2⋅(−3)⋅(−1)+1⋅5⋅1+(−4)⋅0⋅6−(−4)⋅(−3)⋅1−1⋅(−1)⋅6−2⋅5⋅0=6+5−12+6=5=2cdot(-3)cdot(-1)+1cdot5cdot1+(-4)cdot0cdot6-(-4)cdot(-3)cdot1-1cdot(-1)cdot6-2cdot5cdot0=6+5-12+6=5.

При вычислении определителей таким способом можно легко совершить ошибку из-за невнимательности. Чтобы избежать таких ошибок существует второй способ, называемый правилом Саррюса, или способом «параллельных полосок».

Вычисление определителей по правилу Саррюса

Правило Саррюса также именуют способом присоединения двух строк/столбцов или правилом параллельных полосок.

Основная идея этого правила состоит в приписывании первого и второго столбца справа от определителя.

Вычисления будем производить по следующей схеме:

Как вычислить определитель матрицы третьего порядка 2.png

Перемножаем элементы, соединенные прямыми. Данные произведения берем со знаком «+», если диагональ, на которой они стоят, является главной или параллельной ей; со знаком «-», если она является второй (побочной) или параллельной ей.

Произведения, которые берутся со знаком «+» Произведения, которые берутся со знаком «-»
a11⋅a22⋅a33a_{11} cdot a_{22} cdot a_{33} a13⋅a22⋅a31a_{13}cdot a_{22}cdot a_{31}
a12⋅a23⋅a31a_{12} cdot a_{23} cdot a_{31} a11⋅a23⋅a32a_{11}cdot a_{23}cdot a_{32}
a13⋅a21⋅a32a_{13} cdot a_{21} cdot a_{32} a12⋅a21⋅a33a_{12}cdot a_{21}cdot a_{33}

В общем виде вычисление по правилу Саррюса можно записать следующим образом:

∣a11a12a13a21a22a23a31a32a33∣=∣a11a12a13a21a22a23a31a32a33∣a11a12a21a22a31a32=begin{vmatrix}a_{11}&a_{12}&a_{13}\a_{21}&a_{22}&a_{23}\a_{31}&a_{32}&a_{33}end{vmatrix}=begin{vmatrix}a_{11}&a_{12}&a_{13}\a_{21}&a_{22}&a_{23}\a_{31}&a_{32}&a_{33}end{vmatrix}begin{matrix}a_{11}&a_{12}\a_{21}&a_{22}\a_{31}&a_{32}end{matrix}=

=a11⋅a22⋅a33+a12⋅a23⋅a31+a13⋅a21⋅a32−a13⋅a22⋅a31−a11⋅a23⋅a32−a12⋅a21⋅a33=a_{11}cdot a_{22}cdot a_{33}+a_{12}cdot a_{23}cdot a_{31}+a_{13}cdot a_{21}cdot a_{32}-a_{13}cdot a_{22}cdot a_{31}-a_{11}cdot a_{23}cdot a_{32}-a_{12}cdot a_{21}cdot a_{33}.

Сравнивая эти два способа вычисления определителей, видим одинаковые множители, которые во втором случае немного переставлены местами.
Возможность допустить ошибку, вычисляя определитель по правилу Саррюса, намного меньше.

Примеры

Пример 1

Найти определитель ∣925148637∣begin{vmatrix}9&2&5\1&4&8\6&3&7end{vmatrix} по правилу Саррюса.

Приписываем два первых столбца справа от определителя и вычисляем его:

∣925148637∣921463=begin{vmatrix}9&2&5\1&4&8\6&3&7end{vmatrix}begin{matrix}9&2\1&4\6&3end{matrix}=

=9⋅4⋅7+2⋅8⋅6+5⋅1⋅3−5⋅4⋅6−9⋅8⋅3−2⋅1⋅7=252+96+15−120−216−14=13=9cdot4cdot7+2cdot8cdot6+5cdot1cdot3-5cdot4cdot6-9cdot8cdot3-2cdot1cdot7=252+96+15-120-216-14=13.

Пример 2

Найти определитель ∣21−46−3510−1∣begin{vmatrix}2&1&-4\6&-3&5\1&0&-1end{vmatrix} по правилу Саррюса.

Приписываем два первых столбца справа от определителя и вычисляем его:

∣21−46−3510−1∣216−310=begin{vmatrix}2&1&-4\6&-3&5\1&0&-1end{vmatrix}begin{matrix}2&1\6&-3\1&0end{matrix}=

=2⋅(−3)⋅(−1)+1⋅5⋅1+(−4)⋅6⋅0−(−4)⋅(−3)⋅1−2⋅5⋅0−1⋅6⋅(−1)=6+5−12+6=5=2cdot(-3)cdot(-1)+1cdot5cdot1+(-4)cdot6cdot0-(-4)cdot(-3)cdot1-2cdot5cdot0-1cdot6cdot(-1)=6+5-12+6=5.

Существует еще одна вариация правила Саррюса. Она состоит в приписывании первой и второй строки снизу от определителя. Вычисления производятся аналогично.

Минор и алгебраическое дополнение

Прежде чем перейти к рассмотрению еще одного способа вычисления определителей 3-го порядка разберем 2 понятия: минор, алгебраическое дополнение.

Минор

Минор

Минором MijM_{ij} к элементу aija_{ij} определителя n-го порядка называется определитель (n−1)(n-1)-го порядка, который получается из исходного определителя вычеркиванием ii-той строки и jj-того столбца.

Таким образом, минор — это определитель, который остается после вычеркивания определенной строки и определенного столбца. Например, M11M_{11} получается вычеркиванием 1-й строки и 1-го столбца, M23M_{23} — вычеркиванием 2-й строки и 3-го столбца.

Алгоритм нахождения миноров:

  1. вычеркиваем ii-ю строку;
  2. вычеркиваем jj-й столбец;
  3. записываем определитель, который получили в результате действий 1 и 2.

Примеры

Пример 1

Найти миноры матрицы F=(925148637)F=begin{pmatrix}9&2&5\1&4&8\6&3&7end{pmatrix}.

Те строки и столбцы, которые будем вычеркивать, обозначим зеленым цветом.

M11=∣925148637∣=∣4837∣=4⋅7−3⋅8=28−24=4M_{11}=begin{vmatrix}color{green}9&color{green}2&color{green}5\color{green}1&4&8\color{green}6&3&7end{vmatrix}=begin{vmatrix}4&8\3&7end{vmatrix}=4cdot7-3cdot8=28-24=4,

M12=∣925148637∣=∣1867∣=1⋅7−6⋅8=7−48=−41M_{12}=begin{vmatrix}color{green}9&color{green}2&color{green}5\1&color{green}4&8\6&color{green}3&7end{vmatrix}=begin{vmatrix}1&8\6&7end{vmatrix}=1cdot7-6cdot8=7-48=-41,

M13=∣925148637∣=∣1463∣=1⋅3−6⋅4=3−24=−21M_{13}=begin{vmatrix}color{green}9&color{green}2&color{green}5\1&4&color{green}8\6&3&color{green}7end{vmatrix}=begin{vmatrix}1&4\6&3end{vmatrix}=1cdot3-6cdot4=3-24=-21,

M21=∣925148637∣=∣2537∣=2⋅7−3⋅5=14−15=−1M_{21}=begin{vmatrix}color{green}9&2&5\color{green}1&color{green}4&color{green}8\color{green}6&3&7end{vmatrix}=begin{vmatrix}2&5\3&7end{vmatrix}=2cdot7-3cdot5=14-15=-1,

M22=∣925148637∣=∣9567∣=9⋅7−6⋅5=63−30=33M_{22}=begin{vmatrix}9&color{green}2&5\color{green}1&color{green}4&color{green}8\6&color{green}3&7end{vmatrix}=begin{vmatrix}9&5\6&7end{vmatrix}=9cdot7-6cdot5=63-30=33,

M23=∣925148637∣=∣9263∣=9⋅3−6⋅2=27−12=15M_{23}=begin{vmatrix}9&2&color{green}5\color{green}1&color{green}4&color{green}8\6&3&color{green}7end{vmatrix}=begin{vmatrix}9&2\6&3end{vmatrix}=9cdot3-6cdot2=27-12=15,

M31=∣925148637∣=∣2548∣=2⋅8−4⋅5=16−20=−4M_{31}=begin{vmatrix}color{green}9&2&5\color{green}1&4&8\color{green}6&color{green}3&color{green}7end{vmatrix}=begin{vmatrix}2&5\4&8end{vmatrix}=2cdot8-4cdot5=16-20=-4,

M32=∣925148637∣=∣9518∣=9⋅8−1⋅5=72−5=67M_{32}=begin{vmatrix}9&color{green}2&5\1&color{green}4&8\color{green}6&color{green}3&color{green}7end{vmatrix}=begin{vmatrix}9&5\1&8end{vmatrix}=9cdot8-1cdot5=72-5=67,

M33=∣925148637∣=∣9214∣=9⋅4−1⋅2=36−2=34M_{33}=begin{vmatrix}9&2&color{green}5\1&4&color{green}8\color{green}6&color{green}3&color{green}7end{vmatrix}=begin{vmatrix}9&2\1&4end{vmatrix}=9cdot4-1cdot2=36-2=34.

Пример 2

Найти миноры матрицы G=(21−46−3510−1)G=begin{pmatrix}2&1&-4\6&-3&5\1&0&-1end{pmatrix}.

Те строки и столбцы, которые будем вычеркивать, обозначим зеленым цветом.

M11=∣21−46−3510−1∣=∣−350−1∣=(−3)⋅(−1)−0⋅5=3−0=3M_{11}=begin{vmatrix}color{green}2&color{green}1&color{green}-4\color{green}6&-3&5\color{green}1&0&-1end{vmatrix}=begin{vmatrix}-3&5\0&-1end{vmatrix}=(-3)cdot(-1)-0cdot5=3-0=3,

M12=∣21−46−3510−1∣=∣651−1∣=6⋅(−1)−1⋅5=−6−5=−11M_{12}=begin{vmatrix}color{green}2&color{green}1&color{green}-4\6&color{green}-3&5\1&color{green}0&-1end{vmatrix}=begin{vmatrix}6&5\1&-1end{vmatrix}=6cdot(-1)-1cdot5=-6-5=-11,

M13=∣21−46−3510−1∣=∣6−310∣=6⋅0−1⋅(−3)=0+3=3M_{13}=begin{vmatrix}color{green}2&color{green}1&color{green}-4\6&-3&color{green}5\1&0&color{green}-1end{vmatrix}=begin{vmatrix}6&-3\1&0end{vmatrix}=6cdot0-1cdot(-3)=0+3=3,

M21=∣21−46−3510−1∣=∣1−40−1∣=1⋅(−1)−0⋅(−4)=−1−0=−1M_{21}=begin{vmatrix}color{green}2&1&-4\color{green}6&color{green}-3&color{green}5\color{green}1&0&-1end{vmatrix}=begin{vmatrix}1&-4\0&-1end{vmatrix}=1cdot(-1)-0cdot(-4)=-1-0=-1,

M22=∣21−46−3510−1∣=∣2−41−1∣=2⋅(−1)−1⋅(−4)=−2+4=2M_{22}=begin{vmatrix}2&color{green}1&-4\color{green}6&color{green}-3&color{green}5\1&color{green}0&-1end{vmatrix}=begin{vmatrix}2&-4\1&-1end{vmatrix}=2cdot(-1)-1cdot(-4)=-2+4=2,

M23=∣21−46−3510−1∣=∣2110∣=2⋅0−1⋅1=0−1=−1M_{23}=begin{vmatrix}2&1&color{green}-4\color{green}6&color{green}-3&color{green}5\1&0&color{green}-1end{vmatrix}=begin{vmatrix}2&1\1&0end{vmatrix}=2cdot0-1cdot1=0-1=-1,

M31=∣21−46−3510−1∣=∣1−4−35∣=1⋅5−(−3)⋅(−4)=5−12=−7M_{31}=begin{vmatrix}color{green}2&1&-4\color{green}6&-3&5\color{green}1&color{green}0&color{green}-1end{vmatrix}=begin{vmatrix}
1&-4\-3&5end{vmatrix}=1cdot5-(-3)cdot(-4)=5-12=-7
,

M32=∣21−46−3510−1∣=∣2−465∣=2⋅5−6⋅(−4)=10+24=34M_{32}=begin{vmatrix}2&color{green}1&-4\6&color{green}-3&5\color{green}1&color{green}0&color{green}-1end{vmatrix}=begin{vmatrix}2&-4\6&5end{vmatrix}=2cdot5-6cdot(-4)=10+24=34,

M33=∣21−46−3510−1∣=∣216−3∣=2⋅(−3)−6⋅1=−6−6=−12M_{33}=begin{vmatrix}2&1&color{green}-4\6&-3&color{green}5\color{green}1&color{green}0&color{green}-1end{vmatrix}=begin{vmatrix}2&1\6&-3end{vmatrix}=2cdot(-3)-6cdot1=-6-6=-12.

Алгебраическое дополнение

Алгебраическое дополнение

Алгебраическим дополнением AijA_{ij} к элементу aija_{ij} определителя nn-го порядка называется число Aij=(−1)i+j⋅MijA_{ij}=(-1)^{i+j}cdot M_{ij},

где ii, jj — соответствующие строка и столбец,

MijM_{ij} — минор к элементу aija_{ij}.

Алгоритм нахождения алгебраических дополнений:

  1. найти сумму номеров строки (i)(i) и столбца (j)(j);
  2. найти минор MijM_{ij} по алгоритму нахождения миноров, который представлен выше;
  3. подставить значения, полученные на шагах 1 и 2, в формулу Aij=(−1)i+j⋅MijA_{ij}=(-1)^{i+j}cdot M_{ij}.

Примеры

Пример 1

Найти алгебраические дополнения матрицы F=(925148637)F=begin{pmatrix}9&2&5\1&4&8\6&3&7end{pmatrix}.

A11=(−1)1+1⋅M11=(−1)2⋅∣4837∣=4A_{11}=(-1)^{1+1}cdot M_{11}= (-1)^{2}cdotbegin{vmatrix}4&8\3&7end{vmatrix}=4,

A12=(−1)1+2⋅M12=(−1)3⋅∣1867∣=41A_{12}=(-1)^{1+2}cdot M_{12}= (-1)^{3}cdotbegin{vmatrix}1&8\6&7end{vmatrix}=41,

A13=(−1)1+3⋅M13=(−1)4⋅∣1463∣=−21A_{13}=(-1)^{1+3}cdot M_{13}= (-1)^{4}cdotbegin{vmatrix}1&4\6&3end{vmatrix}=-21,

A21=(−1)2+1⋅M21=(−1)3⋅∣2537∣=1A_{21}=(-1)^{2+1}cdot M_{21}= (-1)^{3}cdotbegin{vmatrix}2&5\3&7end{vmatrix}=1,

A22=(−1)2+2⋅M22=(−1)4⋅∣9567∣=33A_{22}=(-1)^{2+2}cdot M_{22}= (-1)^{4}cdotbegin{vmatrix}9&5\6&7end{vmatrix}=33,

A23=(−1)2+3⋅M23=(−1)5⋅∣9263∣=−15A_{23}=(-1)^{2+3}cdot M_{23}= (-1)^{5}cdotbegin{vmatrix}9&2\6&3end{vmatrix}=-15,

A31=(−1)3+1⋅M31=(−1)4⋅∣2548∣=−4A_{31}=(-1)^{3+1}cdot M_{31}=(-1)^{4}cdotbegin{vmatrix}2&5\4&8end{vmatrix}=-4,

A32=(−1)3+2⋅M32=(−1)5⋅∣9518∣=−67A_{32}=(-1)^{3+2}cdot M_{32}=(-1)^{5}cdotbegin{vmatrix}9&5\1&8end{vmatrix}=-67,

A33=(−1)3+3⋅M33=(−1)6⋅∣9214∣=34A_{33}=(-1)^{3+3}cdot M_{33}=(-1)^{6}cdotbegin{vmatrix}9&2\1&4end{vmatrix}=34.

Пример 2

Найти алгебраические дополнения матрицы G=(21−46−3510−1)G=begin{pmatrix}2&1&-4\6&-3&5\1&0&-1end{pmatrix}.

A11=(−1)1+1⋅M11=(−1)2⋅∣−350−1∣=3A_{11}=(-1)^{1+1}cdot M_{11}=(-1)^{2}cdotbegin{vmatrix}-3&5\0&-1end{vmatrix}=3,

A12=(−1)1+2⋅M12=(−1)3⋅∣651−1∣=11A_{12}=(-1)^{1+2}cdot M_{12}=(-1)^{3}cdotbegin{vmatrix}6&5\1&-1end{vmatrix}=11,

A13=(−1)1+3⋅M13=(−1)4⋅∣6−310∣=3A_{13}=(-1)^{1+3}cdot M_{13}=(-1)^{4}cdotbegin{vmatrix}6&-3\1&0end{vmatrix}=3,

A21=(−1)2+1⋅M21=(−1)3⋅∣1−40−1∣=1A_{21}=(-1)^{2+1}cdot M_{21}=(-1)^{3}cdotbegin{vmatrix}1&-4\0&-1end{vmatrix}=1,

A22=(−1)2+2⋅M22=(−1)4⋅∣2−41−1∣=2A_{22}=(-1)^{2+2}cdot M_{22}=(-1)^{4}cdotbegin{vmatrix}2&-4\1&-1end{vmatrix}=2,

A23=(−1)2+3⋅M23=(−1)5⋅∣2110∣=1A_{23}=(-1)^{2+3}cdot M_{23}=(-1)^{5}cdotbegin{vmatrix}2&1\1&0end{vmatrix}=1,

A31=(−1)3+1⋅M31=(−1)4⋅∣1−4−35∣=−7A_{31}=(-1)^{3+1}cdot M_{31}=(-1)^{4}cdotbegin{vmatrix}1&-4\-3&5end{vmatrix}=-7,

A32=(−1)3+2⋅M32=(−1)5⋅∣2−465∣=−34A_{32}=(-1)^{3+2}cdot M_{32}=(-1)^{5}cdotbegin{vmatrix}2&-4\6&5end{vmatrix}=-34,

A33=(−1)3+3⋅M33=(−1)6⋅∣216−3∣=−12A_{33}=(-1)^{3+3}cdot M_{33}=(-1)^{6}cdotbegin{vmatrix}2&1\6&-3end{vmatrix}=-12.

Зная, что такое миноры и алгебраические дополнения, рассмотрим вычисление определителя по строке и столбцу.

Вычисление определителя по строке или столбцу

Определитель матрицы равен сумме произведений элементов строки (столбца) на их алгебраические дополнения.

Алгоритм вычисления определителя по строке или столбцу:

  1. находим алгебраические дополнения элементов строки или столбца;
  2. находим произведения элементов на их алгебраические дополнения;
  3. находим сумму, полученных на шаге 2, произведений.

Примеры

Пример 1

Найти определитель ∣925148637∣begin{vmatrix}9&2&5\1&4&8\6&3&7end{vmatrix} по 2 столбцу.

∣925148637∣=2⋅A12+4⋅A22+3⋅begin{vmatrix}9&2&5\1&4&8\6&3&7end{vmatrix}=2cdot A_{12}+4cdot A_{22}+3cdot

A32=2(−1)3M12+4(−1)4M22+3(−1)5M32=2(−1)3∣1867∣+4(−1)4∣9567∣+3(−1)5∣9518∣=A_{32}=2(-1)^{3}M_{12}+4(-1)^{4}M_{22}+3(-1)^{5}M_{32}=2(-1)^{3}begin{vmatrix}1&8\6&7end{vmatrix}+4(-1)^{4}begin{vmatrix}9&5\6&7end{vmatrix}+3(-1)^{5}begin{vmatrix}9&5\1&8end{vmatrix}=

=−2⋅(−41)+4⋅33−3⋅67=82+132−201=13=-2cdot(-41)+4cdot33-3cdot67=82+132-201=13.

Пример 2

Найти определитель ∣21−46−3510−1∣begin{vmatrix}2&1&-4\6&-3&5\1&0&-1end{vmatrix} по 3 строке.

∣21−46−3510−1∣=1⋅A31+0⋅A32−1⋅A33=1(−1)4M31+0(−1)5M32−1(−1)6M33=begin{vmatrix}2&1&-4\6&-3&5\1&0&-1end{vmatrix}=1cdot A_{31}+0cdot A_{32}-1cdot A_{33}=1(-1)^{4}M_{31}+0(-1)^{5}M_{32}-1(-1)^{6}M_{33}=

=1(−1)4∣1−4−35∣+0(−1)5∣2−465∣−1(−1)6∣216−3∣=−7+0+12=5=1(-1)^{4}begin{vmatrix}1&-4\-3&5end{vmatrix}+0(-1)^{5}begin{vmatrix}2&-4\6&5end{vmatrix}-1(-1)^{6}begin{vmatrix}2&1\6&-3end{vmatrix}=-7+0+12=5.

Любой из рассмотренных способов можно применять при нахождении определителей третьего порядка. В следующий раз мы разберем вычисление определителей матриц высших порядков.

Оформите решение задачи на заказ онлайн, если возникают трудности с выполнением!

Тест по теме «Как вычислить определитель матрицы третьего порядка»

Содержание:

Определители II и III порядка

Определение: Определителем порядка n называется число (выражение), записанное в виде квадратной таблицы, имеющей n строк и n столбцов, которая раскрывается по определенному правилу.

Определитель матрицы - определение и вычисление с примерами решения

Числа Определитель матрицы - определение и вычисление с примерами решения

Определение: Определителем II порядка называется число (выражение), записанное в виде квадратной таблицы размером 2×2, т.е. имеющая 2 строки и 2 столбца.

Определение: Определитель II порядка вычисляется по правилу: из произведения элементов, стоящих на главной диагонали, надо вычесть произведение элементов, стоящих на побочной диагонали: Определитель матрицы - определение и вычисление с примерами решения

Пример:

Определитель матрицы - определение и вычисление с примерами решения

Определение: Определителем III порядка называется число (выражение), записанное в виде квадратной таблицы размером 3×3, то есть имеющей 3 строки и 3 столбца.

Определитель III порядка вычисляется по правилу Саррюса: за определителем выписывают первый и второй столбцы, затем из суммы произведений элементов, стоящих на главной диагонали ей параллельных диагоналях, надо вычесть сумму произведений элементов, стоящих на побочной диагонали и ей параллельных: Определитель матрицы - определение и вычисление с примерами решения

Определитель матрицы - определение и вычисление с примерами решения

Пример:

Определитель матрицы - определение и вычисление с примерами решения

Определение: Минором Определитель матрицы - определение и вычисление с примерами решения элемента Определитель матрицы - определение и вычисление с примерами решения называется определитель порядка (n-1), который получается из исходного определителя порядка n путем вычеркивания строки i и столбца j, на пересечении которых стоит элемент Определитель матрицы - определение и вычисление с примерами решения

Определитель матрицы - определение и вычисление с примерами решения

Пример:

Найти миноры элементов Определитель матрицы - определение и вычисление с примерами решенияи Определитель матрицы - определение и вычисление с примерами решенияопределителя из Примера 2. Вычеркивая в определителе строку 1 и столбец 2:Определитель матрицы - определение и вычисление с примерами решения получим минорОпределитель матрицы - определение и вычисление с примерами решения Поступая аналогично со строкой 3 и столбцом 3, получим минор Определитель матрицы - определение и вычисление с примерами решения

Пример:

Найти миноры элементов Определитель матрицы - определение и вычисление с примерами решения и Определитель матрицы - определение и вычисление с примерами решения определителя Определитель матрицы - определение и вычисление с примерами решения Исходя из определения минора Определитель матрицы - определение и вычисление с примерами решения получаем Определитель матрицы - определение и вычисление с примерами решения аналогично найдем минор Определитель матрицы - определение и вычисление с примерами решения

Определение: Алгебраическим дополнением Определитель матрицы - определение и вычисление с примерами решенияэлемента Определитель матрицы - определение и вычисление с примерами решения называется произведение минора этого элемента на Определитель матрицы - определение и вычисление с примерами решения т.е. Определитель матрицы - определение и вычисление с примерами решения

Замечание: Из определения алгебраического дополнения следует, что алгебраическое дополнение совпадает со своим минором, если сумма Определитель матрицы - определение и вычисление с примерами решения является четным числом, и противоположно ему по знаку, если сумма Определитель матрицы - определение и вычисление с примерами решения – нечетное число.

Определение: Транспонированным определителем n-го порядка называется определитель порядка n, полученный из исходного определителя путем замены строк на соответствующие столбцы, а столбцов на соответствующие строки.

Если Определитель матрицы - определение и вычисление с примерами решения

Пример:

Найти определитель, транспонированный к определителюОпределитель матрицы - определение и вычисление с примерами решения Из определения транспонированного определителя Определитель матрицы - определение и вычисление с примерами решения

Свойства определителей

1. Величина транспонированного определителя равна величине исходного определителя. Пусть Определитель матрицы - определение и вычисление с примерами решения Отсюда видно, что Определитель матрицы - определение и вычисление с примерами решения

2. Перестановка местами двух строк (столбцов) изменяет знак определителя на противоположный. Пусть Определитель матрицы - определение и вычисление с примерами решения

Определитель матрицы - определение и вычисление с примерами решения

Если поменять местами строки (столбцы) четное число раз, то величина и знак определителя не меняется. Нечетная перестановка местами строк (столбцов) не меняет величину определителя, но изменяет его знак на противоположный.

3. Определитель, содержащий две (или более) одинаковых строки (столбца), равен нулю. Если определитель содержит два одинаковых столбца, то Определитель матрицы - определение и вычисление с примерами решения Определитель матрицы - определение и вычисление с примерами решения

4. Для того чтобы умножить определитель на число k, достаточно умножить на это число все элементы какой-либо одной строки (столбца). Обратно: если все элементы какой-либо строки (столбца) имеют общий множитель k, то его можно вынести за знак определителя.

Докажем это свойство: Определитель матрицы - определение и вычисление с примерами решения

5. Если две каких-либо строки (столбца) пропорциональны, то определитель равен нулю.

Пусть в определителе II порядка первая и вторая строки пропорциональны, тогда Определитель матрицы - определение и вычисление с примерами решения

6. Если все элементы какой-либо строки (столбца) равны нулю, то определитель равен нулю.

Пусть в определителе II порядка все элементы первой строки равны нулю, тогда Определитель матрицы - определение и вычисление с примерами решения

7. Если элементы какой-либо строки (или столбца) можно представить в виде двух слагаемых, то сам определитель можно представить в виде суммы двух определителей. Если Определитель матрицы - определение и вычисление с примерами решения Доказать самостоятельно.

8. Если все элементы какой-либо строки (столбца) умножить на вещественное число к и прибавить k соответствующим элементам другой строки (соответственно, столбца), то величина определителя не изменится.

Умножим элементы второго столбца на вещественное число k и прибавим результат умножения к соответствующим элементам первого столбца, получимОпределитель матрицы - определение и вычисление с примерами решения

Второй определитель равен нулю по свойству 5.

Замечание: Данное свойство применяется для обнуления всех элементов какой-либо строки (столбца) за исключением одного (метод обнуления), что существенно снижает трудоемкость вычисления определителей порядка выше 3 (см. также свойство 9.).

9. [Метод раскрытия определителя по элементам какой-либо строки (или столбца); универсальный способ вычисления определителя любого порядка]. Определитель любого порядка равен сумме произведений элементов какой-либо строки (столбца) на их алгебраические дополнения:

Определитель матрицы - определение и вычисление с примерами решения

Пример:

Вычислить определитель Определитель матрицы - определение и вычисление с примерами решения по элементам 3 строки и по элементам 2 столбца.

Решение:

Воспользуемся свойством 9.: раскроем определитель по элементам 3 строки Определитель матрицы - определение и вычисление с примерами решения Вычислим определитель по элементам 2 столбцаОпределитель матрицы - определение и вычисление с примерами решения

Определитель матрицы - определение и вычисление с примерами решения

Из полученных результатов видно, что свойство 9. является универсальным методом вычисления любых определителей по элементам любой строки или столбца.

Используя свойство 8. можно обнулить все элементы какой-либо строки (столбца) за исключением одного (метод обнуления), а затем раскрыть определитель по элементам этой строки, воспользовавшись свойством 9.

Пример:

Вычислить определитель Определитель матрицы - определение и вычисление с примерами решения

Решение:

Обнулим элементы в третьей строке, для чего выполним следующие действия: Определитель матрицы - определение и вычисление с примерами решения (по свойству 4. из третьей строки вынесем множитель 2) Определитель матрицы - определение и вычисление с примерами решенияиспользуя свойство 8., умножим все элементы второго столбца на 1.5 и прибавим к соответствующим элементам третьего столбца, получим) Определитель матрицы - определение и вычисление с примерами решения

(по свойству 4. из третьего столбца вынесем множитель 0,5, тогда множитель перед определителем станет равным 1) Определитель матрицы - определение и вычисление с примерами решения

(раскроем определитель по элементам третьей строки: Определитель матрицы - определение и вычисление с примерами решениявыше из определителя третьего порядка вычеркнута третья строка с нулями и второй столбец, т.е. показан необходимый для дальнейших вычислений минор Определитель матрицы - определение и вычисление с примерами решения Таким образом, метод обнуления позволяет значительно ускорить процесс вычисления любого определителя.

Пример:

Решить уравнение Определитель матрицы - определение и вычисление с примерами решения

Решение:

Вычислим определители второго и третьего порядков согласно вышеописанным правилам:

Определитель матрицы - определение и вычисление с примерами решения

Найденные величины подставим в исходное уравнение

Определитель матрицы - определение и вычисление с примерами решения

Пример:

Решить неравенство Определитель матрицы - определение и вычисление с примерами решения

Решение:

Вычислим определители второго и третьего порядков согласно вышеописанным правилам:Определитель матрицы - определение и вычисление с примерами решения

Определитель матрицы - определение и вычисление с примерами решения

Найденные величины подставим в исходное неравенство Определитель матрицы - определение и вычисление с примерами решения

Пример:

Вычислить определитель четвертого порядка (аналогично выполнить такие же действия с определителем третьего порядка), преобразовав его так, чтобы три элемента некоторого ряда равнялись нулю, и вычислить полученный определитель по элементам этого ряда: Определитель матрицы - определение и вычисление с примерами решения

Решение:

Во второй строке исходного определителя присутствуют 1 и 0, поэтому обнуление элементов будем производить в этой строке (при обнулении элементов в строке действия производят со столбцами и наоборот): Определитель матрицы - определение и вычисление с примерами решения – строка обнуления; Определитель матрицы - определение и вычисление с примерами решения– столбцы, с которыми производят действия)=Определитель матрицы - определение и вычисление с примерами решения

(по методу обнуления раскроем определитель по элементам 2-ой строки (Определитель матрицы - определение и вычисление с примерами решения – цифры, с которыми производятся действия))Определитель матрицы - определение и вычисление с примерами решения

Определитель матрицы - определение и вычисление с примерами решения

Определитель матрицы - определение и вычисление с примерами решения (по универсальному методу раскроем определитель по элементам третьей строки)Определитель матрицы - определение и вычисление с примерами решения

Определитель матрицы - определение и вычисление с примерами решения

Определители

Перестановкой чисел 1, 2,…, n называется любое расположение этих чисел в определенном порядке. В элементарной алгебре доказывается, что число всех перестановок, которые можно образовать из n чисел, равно 12…n = n!. Например, из трех чисел 1, 2, 3 можно образовать 3!=6 перестановок: 123, 132, 312, 321, 231, 213. Говорят, что в данной перестановке числа i и j составляют инверсию (беспорядок), если i>j, но i стоит в этой перестановке раньше j, то есть если большее число стоит левее меньшего.

Перестановка называется четной (или нечетной), если в ней соответственно четно (нечетно) общее число инверсий. Операция, посредством которой от одной перестановки переходят к другой, составленной из тех же n чисел, называется подстановкой n-ой степени.

Подстановка, переводящая одну перестановку в другую, записывается двумя строками в общих скобках, причем числа, занимающие одинаковые места в рассматриваемых перестановках, называются соответствующими и пишутся одно под другим. Например, символ Определитель матрицы - определение и вычисление с примерами решения обозначает подстановку в которой 3 переходит в Определитель матрицы - определение и вычисление с примерами решения

Подстановка называется четной (или нечетной), если общее число инверсий в обеих строках подстановки четно (нечетно). Всякая подстановка n-ой степени может быть записана в виде Определитель матрицы - определение и вычисление с примерами решения т.е. с натуральным расположением чисел в верхней строке.

Пусть нам дана квадратная матрица порядка n Определитель матрицы - определение и вычисление с примерами решения

Рассмотрим все возможные произведения по n элементов этой матрицы, взятых по одному и только по одному из каждой строки и каждого столбца, т.е. произведений вида: Определитель матрицы - определение и вычисление с примерами решения

где индексы Определитель матрицы - определение и вычисление с примерами решения составляют некоторую перестановку из чисел 1, 2,…,n. Число таких произведений равно числу различных перестановок из n символов, т.е. равно n!. Знак произведения (4.4) равен (-1)q где q – число инверсий в перестановке вторых индексов элементов.

Определителем n-го порядка, соответствующим матрице (4.3), называется алгебраическая сумма n! членов вида (4.4). Для записи определителя употребляется символ Определитель матрицы - определение и вычисление с примерами решения (детерминант, или определитель, матрицы А).

Свойства определителей:

  1. Определитель не меняется при транспонировании.
  2. Если одна из строк определителя состоит из нулей, то определитель равен нулю.
  3. Если в определителе переставить две строки, определитель поменяет знак.
  4. Определитель, содержащий две одинаковые строки, равен нулю.
  5. Если все элементы некоторой строки определителя умножить на некоторое число Определитель матрицы - определение и вычисление с примерами решения то сам определитель умножится на Определитель матрицы - определение и вычисление с примерами решения
  6. Определитель, содержащий две пропорциональные строки, равен нулю.
  7. Если все элементы i-й строки определителя представлены в виде суммы двух слагаемых Определитель матрицы - определение и вычисление с примерами решения то определитель равен сумме определителей, у которых все строки, кроме i-ой, – такие же, как в заданном определителе, а i-я строка в одном из слагаемых состоит из элементов Определитель матрицы - определение и вычисление с примерами решения в другом – из элементов Определитель матрицы - определение и вычисление с примерами решения
  8. Определитель не меняется, если к элементам одной из его строк прибавляются соответствующие элементы другой строки, умноженные на одно и то же число.

Замечание. Все свойства остаются справедливыми, если вместо строк взять столбцы.

Минором Определитель матрицы - определение и вычисление с примерами решения элемента Определитель матрицы - определение и вычисление с примерами решения определителя d n-го порядка называется определитель порядка n-1, который получается из d вычеркиванием строки и столбца, содержащих данный элемент.

Алгебраическим дополнением элемента Определитель матрицы - определение и вычисление с примерами решения определителя d называется его минор Определитель матрицы - определение и вычисление с примерами решения взятый со знаком Определитель матрицы - определение и вычисление с примерами решения Алгебраическое дополнение элемента Определитель матрицы - определение и вычисление с примерами решения будем обозначать Определитель матрицы - определение и вычисление с примерами решения Таким образом, Определитель матрицы - определение и вычисление с примерами решения

Способы практического вычисления определителей, основанные на том, что определитель порядка n может быть выражен через определители более низких порядков, дает следующая теорема.

  • Заказать решение задач по высшей математике

Теорема (разложение определителя по строке или столбцу).

Определитель равен сумме произведений всех элементов произвольной его строки (или столбца) на их алгебраические дополнения. Иначе говоря, имеет место разложение d по элементам i-й строки Определитель матрицы - определение и вычисление с примерами решения или j- го столбца Определитель матрицы - определение и вычисление с примерами решения

В частности, если все элементы строки (или столбца), кроме одного, равны нулю, то определитель равен этому элементу, умноженному на его алгебраическое дополнение.

Пример:

Не вычисляя определителя Определитель матрицы - определение и вычисление с примерами решения показать, что он равен нулю.

Решение:

Вычтем из второй строки первую, получим определитель Определитель матрицы - определение и вычисление с примерами решения равный исходному. Если из третьей строки также вычесть первую, то получится определитель Определитель матрицы - определение и вычисление с примерами решения в котором две строки пропорциональны.

Такой определитель равен нулю.

Пример:

Вычислить определитель Определитель матрицы - определение и вычисление с примерами решения разложив его по элементам второго столбца.

Решение:

Разложим определитель по элементам второго столбца: Определитель матрицы - определение и вычисление с примерами решения

Пример:

Вычислить определитель Определитель матрицы - определение и вычисление с примерами решения в котором все элементы по одну сторону от главной диагонали равны нулю.

Решение:

Разложим определитель А по первой строке:

Определитель матрицы - определение и вычисление с примерами решения

Определитель, стоящий справа, можно снова разложить по первой строке, тогда получим: Определитель матрицы - определение и вычисление с примерами решения

И так далее. После n шагов придем к равенству Определитель матрицы - определение и вычисление с примерами решения

Пример:

Вычислить определитель Определитель матрицы - определение и вычисление с примерами решения

Решение:

Если к каждой строке определителя, начиная со второй, прибавить первую строку, то получится определитель, в котором все элементы, находящиеся ниже главной диагонали, будут равны нулю. А именно, получим определитель: Определитель матрицы - определение и вычисление с примерами решения равный исходному.

Рассуждая, как в предыдущем примере найдем, что он равен произведению элементов главной диагонали, т.е. n!. Способ, с помощью которого вычислен данный определитель, называется способом приведения к треугольному виду.

——- в вышмате

Определители. Алгебраические дополнения

Внимание! Понятие определителя вводится только для квадратной матрицы.

Матрица называется квадратной порядка n, если количество ее строк совпадает с количеством столбцов и равно n.

Элементы квадратной матрицы, имеющие одинаковые значения индексов, составляют главную диагональ. Элементы квадратной матрицы порядка n, сумма индексов каждого из которых равна n+1, составляют побочную диагональ.

Определитель матрицы Определитель матрицы - определение и вычисление с примерами решения обозначается одним из следующих символов: Определитель матрицы - определение и вычисление с примерами решения

Внимание! Определитель – это число, характеризующее квадратную мат- рицу.

Определитель матрицы второго порядка равен разности элементов главной и побочной диагоналей соответственно:

Определитель матрицы - определение и вычисление с примерами решения

Определитель матрицы третьего порядка равен сумме элементов главной диагонали и элементов, расположенных в вершинах треугольников с основаниями, параллельными главной диагонали, а также разности элементов побочной диагонали и элементов, расположенных в вершинах треугольников с основаниями, параллельными побочной диагонали. Определитель матрицы - определение и вычисление с примерами решения

Схематично это правило изображается так (правило треугольника): Определитель матрицы - определение и вычисление с примерами решения

Например,

Определитель матрицы - определение и вычисление с примерами решения Квадратная матрица называется верхней (нижней) треугольной, если все элементы, стоящие под (над) главной диагональю равны нулю.

Отметим некоторые свойства определителя.

  1. Определитель треугольной матрицы равен произведению элементов главной диагонали.
  2. При транспонировании матрицы ее определитель не изменяется.
  3. От перестановки двух рядов (строк или столбцов) определитель меняет знак.
  4. Общий множитель всех элементов некоторого ряда определителя можно выносить за знак определителя.
  5. Если все элементы какого-нибудь ряда матрицы равны нулю, то определитель равен нулю.
  6. Определитель, содержащий два пропорциональных ряда, равен нулю.
  7. Определитель не изменится, если к элементам какого-либо ряда прибавить соответствующие элементы другого ряда, умноженные на одно и то же число.
  8. Определитель произведения двух матриц одинакового порядка равен произведению определителей этих матриц.

Минором элемента Определитель матрицы - определение и вычисление с примерами решения определителя n-го порядка называется определитель (n-l)-ro порядка, получаемый вычеркиванием i-й строки и j-ro столбца, на пересечении которых стоит этот элемент. Обозначение: Определитель матрицы - определение и вычисление с примерами решения

Алгебраическим дополнением элемента Определитель матрицы - определение и вычисление с примерами решения называется его минор, умноженный на Определитель матрицы - определение и вычисление с примерами решения Обозначение: Определитель матрицы - определение и вычисление с примерами решения

Определитель матрицы - определение и вычисление с примерами решения

Теорема разложения.

Определитель матрицы равен сумме произведений элементов любого ряда на их алгебраические дополнения.

Пример №2

Вычислить определитель, разлагая его по элементам первой строки: Определитель матрицы - определение и вычисление с примерами решения

Решение:

По теореме разложения Определитель матрицы - определение и вычисление с примерами решения

Найдем алгебраические дополнения элементов матрицы А: Определитель матрицы - определение и вычисление с примерами решения

Следовательно,

Определитель матрицы - определение и вычисление с примерами решения

Для вычисления определителя порядка выше третьего удобно пользоваться теоремой разложения (метод понижения порядка) или методом приведения определителя к треугольному виду.

Пример №3

Вычислить определитель, приведя его к треугольному виду:

Определитель матрицы - определение и вычисление с примерами решения

Решение:

Применяя свойство 6 определителей, преобразуем последовательно второй, третий, четвертый столбцы матрицы. Определитель матрицы - определение и вычисление с примерами решения

  1. прибавили ко второму столбцу первый, умноженный на -2;
  2. прибавили к третьему столбцу первый, умноженный на -3;
  3. прибавили к четвертому столбцу первый, умноженный на -4;
  4. применили свойство 1 определителей.
  • Критерий совместности Кронекера-Капелли
  • Формулы Крамера
  • Матричный метод
  • Экстремум функции
  • Пределы в математике
  • Функции многих переменных
  • Уравнения прямых и кривых на плоскости
  • Плоскость и прямая в пространстве

Добавить комментарий